Chapter 33 Coronary Artery Disease and Acute Coronary Syndrome - Complex Fall 2021

अब Quizwiz के साथ अपने होमवर्क और परीक्षाओं को एस करें!

Which instruction would the nurse give to a patient about lowering low-density lipoprotein (LDL) cholesterol? Select all that apply. "Avoid fatty fish, such as tuna." "Increase complex carbohydrates, such as fruit." "Fat intake should be about 40% of total calories." "Increase fiber intake by eating food, such as legumes." "Choose foods made with whole grains, such as brown rice."

"Increase complex carbohydrates, such as fruit." "Increase fiber intake by eating food, such as legumes." "Choose foods made with whole grains, such as brown rice." It is recommended to increase the intake of complex carbohydrates, fiber, and whole grains. The American Heart Association (AHA) recommends eating fatty fish twice a week. Fat intake should be about 25% to 35% of total daily calories.

A patient is recovering from an uncomplicated myocardial infarction (MI). Which instructions will the nurse include about physical activity? "Start out with 30-minute sessions." "Be sure to perform physical activity at least twice a week." "Physical activity should be regular, rhythmic, and repetitive." "Your heart rate during exercise should only be 30 beats over your resting rate."

"Physical activity should be regular, rhythmic, and repetitive." Physical activity should be regular, rhythmic, and repetitive, using large muscles to build up endurance (e.g., walking, cycling, swimming, rowing). Physical activity sessions should be at least 30 minutes long. Instruct the patient to begin slowly at personal tolerance (perhaps only 5 to 10 minutes) and build up to 30 minutes. Encourage the patient to perform physical activity on most days of the week. Activity intensity is determined by the patient's heart rate. If an exercise stress test has not been performed, the heart rate of the patient recovering from an MI should not exceed 20 beats/min over the resting heart rate.

Which drug prevents the binding of fibrinogen to platelets, thereby preventing platelet aggregation? 1. Heparin. 2. Tirofiban. 3. Ticagrelor. 4. Dalteparin.

2. Tirofiban Tirofiban is a glycoprotein IIb/IIIa inhibitor, which prevents the biding of fibrinogen to platelets, thereby blocking platelet aggregation. Heparin is an anticoagulant, which prevents conversion of fibrinogen to fibrin and prothrombin to thrombin. Ticagrelor is an antiplatelet agent, which inhibits platelet aggregation. Dalteparin is a low-molecular-weight heparin, which binds to antithrombin III, enhancing its effect.

What risk factor for coronary artery disease is considered nonmodifiable? 1. Obesity. 2. Diabetes. 3. Tobacco use. 4. Family history of heart disease.

4. Family history of heart disease. Family history of heart disease is a nonmodifiable risk factor for coronary artery disease. Diabetes, obesity, and tobacco use are all modifiable risk factors for coronary artery disease.

A patient recently diagnosed with coronary artery disease (CAD) asks the nurse: "What caused my problem?" Which cause would the nurse explain? 1. Orthostatic hypotension. 2. Low oxygen saturation in the blood. 3. The heart's inability to pump effectively. 4. Fatty deposits on the walls of the coronary arteries.

4. Fatty deposits on the walls of the coronary arteries. The major cause of CAD is atherosclerosis, which is manifested by fatty deposits on the walls of coronary arteries. Decrease in pumping of the heart will result in congestive heart failure (CHF). Low oxygen saturation of the blood is a result of respiratory problems. Hypertension, not orthostatic hypotension, will predispose a patient to developing CAD.

When admitting a patient with a non-ST-segment-elevation myocardial infarction (NSTEMI) to the intensive care unit, which action should the nurse perform first? a. Attach the heart monitor. b. Obtain the blood pressure. c. Assess the peripheral pulses. d. Auscultate the breath sounds.

ANS: A Because dysrhythmias are the most common complication of myocardial infarction (MI), the first action should be to place the patient on a heart monitor. The other actions are also important and should be accomplished as quickly as possible.

The nurse is developing a teaching plan for a 64-year-old patient with coronary artery disease (CAD). Which factor should the nurse focus on during the teaching session? a. Family history of coronary artery disease b. Elevated low-density lipoprotein (LDL) level c. Greater risk associated with the patient's gender d. Increased risk of cardiovascular disease with aging

ANS: B Because family history, gender, and age are nonmodifiable risk factors, the nurse should focus on the patient's LDL level. Decreases in LDL will help reduce the patient's risk for developing CAD.

The nurse is caring for a patient who is recovering from a sudden cardiac death (SCD) event and has no evidence of an acute myocardial infarction (AMI). What should the nurse anticipate teaching the patient? a. Sudden cardiac death events rarely reoccur. b. Additional diagnostic testing will be required. c. Long-term anticoagulation therapy will be needed. d. Limiting physical activity will prevent future SCD events.

ANS: B Diagnostic testing (e.g., stress test, Holter monitor, electrophysiologic studies, cardiac catheterization) is used to determine the possible cause of the SCD and treatment options. SCD is likely to recur. Anticoagulation therapy will not have any effect on the incidence of SCD, and SCD can occur even when the patient is resting.

The nurse is caring for a patient who was admitted to the coronary care unit following an acute myocardial infarction (AMI) and percutaneous coronary intervention the previous day. What should teaching for this patient include today? a. Typical emotional responses to AMI b. When cardiac rehabilitation will begin c. Pathophysiology of coronary artery disease d. Information regarding discharge medications

ANS: B Early after an AMI, the patient will want to know when resumption of usual activities can be expected. At this time, the patient's anxiety level or denial will interfere with good understanding of complex information such as the pathophysiology of coronary artery disease. Teaching about discharge medications should be done closer to discharge. The nurse should support the patient by decreasing anxiety rather than discussing the typical emotional responses to myocardial infarction.

After reviewing a patient's history, vital signs, physical assessment, and laboratory data, which information shown in the accompanying figure is most important for the nurse to communicate to the health care provider? a. Hyperglycemia b. Bilateral crackles c. Q waves on ECG d. Elevated troponin

ANS: B Pulmonary congestion suggests that the patient may be developing heart failure, a complication of myocardial infarction (MI). Hyperglycemia is common after MI because of the inflammatory process that occurs with tissue necrosis. Troponin levels will be elevated for several days after MI. Q waves often develop with ST-segment-elevation MI.

Which information about a patient receiving thrombolytic therapy for an acute myocardial infarction is most important for the nurse to communicate to the health care provider? a. An increase in troponin levels from baseline b. A large bruise at the patient's IV insertion site c. No change in the patient's reported level of chest pain d. A decrease in ST-segment elevation on the electrocardiogram

ANS: C Continued chest pain suggests that the thrombolytic therapy is not effective and that other interventions such as percutaneous coronary intervention may be needed. Bruising is a possible side effect of thrombolytic therapy, but it is not an indication that therapy should be discontinued. The decrease of the ST-segment elevation indicates that thrombolysis is occurring, and perfusion is returning to the injured myocardium. An increase in troponin levels is expected with reperfusion and is related to the washout of cardiac biomarkers into the circulation as the blocked vessel is opened.

Heparin is ordered for a patient with a non-ST-segment-elevation myocardial infarction (NSTEMI). How should the nurse explain the purpose of the heparin to the patient? a. "Heparin enhances platelet aggregation at the plaque site." b. "Heparin decreases the size of the coronary artery plaque." c. "Heparin prevents the development of new clots in the coronary arteries." d. "Heparin dissolves clots that are blocking blood flow in the coronary arteries."

ANS: C Heparin helps prevent the conversion of fibrinogen to fibrin and decreases coronary artery thrombosis. It does not change coronary artery plaque, dissolve already formed clots, or enhance platelet aggregation.

After receiving change-of-shift report about the following four patients on the cardiac care unit, which patient should the nurse assess first? a. A 39-year-old patient with pericarditis who is complaining of sharp, stabbing chest pain. b. A 56-year-old patient with variant angina who is scheduled to receive nifedipine (Procardia). c. A 65-year-old patient who had a myocardial infarction (MI) 4 days ago and is anxious about today's planned discharge. d. A 59-year-old patient with unstable angina who has just returned after a percutaneous coronary intervention (PCI).

ANS: D After PCI, the patient is at risk for hemorrhage from the arterial access site. The nurse should assess the patient's blood pressure, pulses, and the access site immediately. The other patients should also be assessed as quickly as possible, but assessment of this patient has the highest priority.

Which information from a patient helps the nurse confirm the previous diagnosis of chronic stable angina? a. "The pain wakes me up at night." b. "The pain is level 3 to 5 (0 to 10 scale)." c. "The pain has gotten worse over the last week." d. "The pain goes away after a nitroglycerin tablet."

ANS: D Chronic stable angina is typically relieved by rest or nitroglycerin administration. The level of pain is not a consistent indicator of the type of angina. Pain occurring at rest or with increased frequency is typical of unstable angina.

Which patient statement indicates that the nurse's teaching about sublingual nitroglycerin (Nitrostat) has been effective? a. "I can expect nausea as a side effect of nitroglycerin." b. "I should only take nitroglycerin when I have chest pain." c. "Nitroglycerin helps prevent a clot from forming and blocking blood flow to my heart." d. "I will call an ambulance if I have pain after taking 3 nitroglycerin 5 minutes apart."

ANS: D The emergency response system (ERS) should be activated when chest pain or other symptoms are not completely relieved after three sublingual nitroglycerin tablets taken 5 minutes apart. Nitroglycerin can be taken to prevent chest pain or other symptoms from developing (e.g., before intercourse). Gastric upset (e.g., nausea) is not an expected side effect of nitroglycerin. Nitroglycerin does not impact the underlying pathophysiology of coronary artery atherosclerosis.

The nurse would assess a patient with reports of chest pain for which clinical manifestations associated with a myocardial infarction (MI)? (Select all that apply.) Flushing Ashen skin Diaphoresis Nausea and vomiting S3 or S4 heart sounds

Ashen skin Diaphoresis Nausea and vomiting S3 or S4 heart sounds Rationale: During the initial phase of an MI, catecholamines are released from the ischemic myocardial cells, causing increased sympathetic nervous system stimulation. This results in the release of glycogen, diaphoresis, and vasoconstriction of peripheral blood vessels. The patient's skin may be ashen, cool, and clammy (not flushed) because of this response. Nausea and vomiting may result from reflex stimulation of the vomiting center by severe pain. Ventricular dysfunction resulting from the MI may lead to the presence of the abnormal S3 and S4 heart sounds.

The nurse is teaching a patient recovering from a myocardial infarction. How should resumption of sexual activity be discussed? 1. Delegated to the primary care provider. 2. Discussed along with other physical activities. 3. Avoided because it is embarrassing to the patient. 4. Accomplished by providing the patient with written material.

Discussed along with other physical activities Rationale: Although some nurses may not feel comfortable discussing sexual activity with patients, it is a necessary component of patient teaching. It is helpful to consider sex as a physical activity and to discuss or explore feelings in this area when other physical activities are discussed. Although providing the patient with written material is appropriate, it should not replace a verbal dialogue that can address the patient's questions and concerns.

Which laboratory result increases a patient's risk for coronary artery disease (CAD)? Decreased triglycerides Elevated low-density lipoproteins (LDLs) Elevated high-density lipoproteins (HDLs) Decreased very-low-density lipoproteins (VLDLs)

Elevated low-density lipoproteins (LDLs) Elevated LDLs contain more cholesterol than any of the other lipoproteins and have an affinity for arterial walls. Elevated LDL levels correlate most closely with an increased incidence of atherosclerosis and CAD. Elevated HDLs, decreased triglycerides, and decreased VLDLs are all negative risk factors for CAD.

The nurse prepares a discharge teaching plan for a patient who has recently been diagnosed with coronary artery disease (CAD). Which priority risk factor should the nurse plan to focus on during the teaching session? Type A personality Elevated serum lipids Family cardiac history High homocysteine levels

Elevated serum lipids Rationale: Dyslipidemia is one of the four major modifiable risk factors for CAD. The other major modifiable risk factors are hypertension, tobacco use, and physical inactivity. Research findings related to psychologic states (i.e., type A personality) as a risk factor for coronary artery disease have been inconsistent. Family history is a nonmodifiable risk factor. High homocysteine levels have been linked to an increased risk for CAD.

Antiplatelet therapy

Low-dose aspirin (81 mg) is recommended for CAD contraindication - history of bleeding -not enough evidence to recommend low-dose aspirin for primary prevention in those younger than 50 or older than 70 Clopidogrel (Plavix) -option for people who are aspirin intolerant

A patient who has received a maximum dose of nitroglycerin continues to report chest pain. Which medication would the nurse anticipate administering to this patient next? Esmolol Docusate Ticagrelor Morphine sulfate

Morphine sulfate Morphine sulfate is the drug of choice for a patient with unrelieved chest pain, even after the administration of nitroglycerin. Esmolol is a β-blocker used to slow down the heart during minimally invasive direct coronary artery bypass (MIDCAB). Docusate is a stool softener that facilitates bowel movements. Ticagrelor is used in dual antiplatelet therapy on a patient with ongoing angina and negative cardiac markers.

Which antilipemic medications should the nurse question for a patient who has cirrhosis of the liver? (Select all that apply.) Niacin Cholestyramine Ezetimibe (Zetia) Gemfibrozil (Lopid) Atorvastatin (Lipitor)

Niacin Ezetimibe (Zetia) Gemfibrozil (Lopid) Atorvastatin (Lipitor) Rationale: Ezetimibe (Zetia) should not be used by patients with liver impairment. Adverse effects of atorvastatin (Lipitor), a statin drug, include liver damage and myopathy. Liver enzymes must be monitored frequently, and the medication stopped if these enzymes increase. Niacin's side effects subside with time, although decreased liver function may occur with high doses. Cholestyramine is safe for long-term use.

PQRST Assessment of Angina

P -Precipitating events -argument -exercise -resting Q - Quality of pain What does it feel like? -pressure -dull -aching -tight -squeezing -heaviness R -Region (location) and radiation of pain -where the pain or discomfort is located? Does it radiate to other areas -back -neck -arms -jaw -shoulder -elbow S - Severity of pain On a scale of 0 to 10, with 0 indicating no pain and 10 being the most severe pain you could imagine, what number would you give the pain or discomfort? T -Timing When did the pain or discomfort begin? Has it changed since this time? Have you had pain/discomfort like this before?

The nurse hears a new murmur at the cardiac apex of a patient with a myocardial infarction (MI). Which complication would the nurse suspect? Acute pericarditis Dressler syndrome Ventricular aneurysm Papillary muscle dysfunction

Papillary muscle dysfunction Papillary muscle dysfunction is a complication of a myocardial infarction and is assessed by the auscultation of a new murmur at the cardiac apex. Acute pericarditis is a complication of a myocardial infarction characterized by the auscultation of a friction rub at the mid- to lower-left sternal border. Dressler syndrome is a complication that develops several weeks after the myocardial infarction, and a pericardial friction rub is auscultated at the mid- to lower-left sternal border. Ventricular aneurysm is a complication of a myocardial infarction without the development of a new murmur or friction rub.

Which patient would experience symptom relief with the administration of nitroglycerin? Patient 1 Patient 2 Patient 3 Patient 4

Patient 1 Patient 1's complaints of chest pressure while walking down the hall align with chronic stable angina and should resolve with the administration of nitroglycerin. Patient 2's midsternal soreness post coronary artery bypass surgery would be treated with pain medication. Patient 3's shortness of breath with heart failure would be treated with diuretics. Patient 4's chest pain related to coronary spasm would be treated with calcium channel blockers.

Which type of angina increases in frequency, duration, and severity over time? Unstable angina Prinzmetal's angina Microvascular angina Chronic stable angina

Unstable angina Unstable angina increases in frequency, duration, and severity as the time progresses. Prinzmetal's angina occurs primarily at rest and is triggered by smoking and increased levels of substances, such as histamine and epinephrine. Microvascular angina is triggered by activities of daily life and exertion. Chronic stable angina is provoked by exertion and relieved by rest.

Collateral circulation

arterial anastomoses or connections Two factors contribute to the growth and extent of collateral circulation: (1) inherited predisposition to develop new blood vessels (angiogenesis) (2) presence of chronic ischemia (poor blood flow) plaque blocks the normal flow of blood through a coronary artery and the resulting ischemia is chronic, increased collateral circulation may develop

Chronic stable angina

chest pain that occurs intermittently over a long period of time with a similar pattern of onset, duration, and intensity of symptoms. provoked by -physical exertion -stress -emotional upset some patients may deny feeling pain, but describe a pressure, heaviness, or discomfort in the chest. -squeezing -heavy -tight -suffocating sensation may be associated with other symptoms, such as dyspnea or fatigue. pain does not change with position or breathing.

Coronary artery disease (CAD)

is a type of blood vessel disorder that we consider in the general category of atherosclerosis. When the atheromas (fatty deposits) form in the coronary arteries. Atherosclerosis is the major cause of CAD.

Complicated lesion

last stage most dangerous lesion fibrous plaque grows, continued inflammation can result in plaque instability, ulceration, and rupture. Once the integrity of the artery's inner wall is compromised (plaque rupture), platelets accumulate in large numbers, leading to a thrombus (platelet aggregation and adhesion is the body's attempt to "heal" the ruptured area). -thrombus may adhere to the wall of the artery, leading to further narrowing or total occlusion of the artery. Activation of the exposed platelets causes expression of glycoprotein Ilb/IIIa receptors that bind fibrinogen. -leads to more platelet aggregation and adhesion, further enlarging the thrombus

Physical inactivity teaching

• Develop and maintain at least 30 minutes of moderate physical activity daily (minimum 5 days a week). • Increase activities to a fitness level.

The nurse provides education to a patient who receives a prescription for sublingual nitroglycerin for treatment of angina. Which statement made by the patient indicates the need for further teaching? "I cannot take sildenafil and nitroglycerin at the same time." "After the bottle is open, the tablets should be replaced every 12 months." "After taking a tablet, I may experience a headache, dizziness, or flushing." "I can take a tablet 5 to 10 minutes before starting an activity that causes angina."

"After the bottle is open, the tablets should be replaced every 12 months." After the bottle is open, the tablets lose potency and should be replaced every 6 months. Patients should be instructed not to combine nitroglycerin with drugs used for erectile dysfunction (e.g., sildenafil [Viagra]) because severe hypotension can occur. Headache, dizziness, or flushing may occur after taking the medication. The patient can take the medication prophylactically before starting an activity that is known to cause angina. In these cases, the patient can take a tablet or spray 5 to 10 minutes before beginning the activity.

The nurse is identifying obese women at risk for developing coronary artery disease (CAD) for possible participation in a health research study. Which criteria are appropriate for the selection of at-risk women? Select all that apply. "Pear"-shaped figure "Apple"-shaped figure Age of less than 40 years Waist circumference more than 30 inches Body mass index (BMI) greater than 30 kg/m2

"Apple"-shaped figure Body mass index (BMI) greater than 30 kg/m2 Obesity is a major risk factor for the development of CAD. Apple-shaped obesity is the type of obesity in which there is more fat deposition around the abdomen. This condition is a major risk factor for development of CAD. Women below the age of 40 are generally premenopausal. The cardioprotective effects of estrogen make premenopausal women less susceptible for developing atherosclerosis, which can lead to CAD. Evidence suggests that people having fat deposition around the thigh and hip regions (pear-shaped figure) are less susceptible to develop coronary artery disease than people having fat deposition around the abdomen (apple-shaped obesity). Obesity in women is defined as having a waist circumference more than 35 inches; therefore the criterion should be "waist circumference greater than 35 inches." Obesity in women is defined as having a body mass index of 30 kg/m2 or greater, which is a major risk factor for development of CAD.

Which suggestion would the nurse provide to a patient with a waist circumference of 42 inches? "Check your blood pressure regularly." "Check your blood glucose levels regularly." "Consume saturated fats for health reasons." "Exercise regularly to maintain ideal weight."

"Exercise regularly to maintain ideal weight." Obesity is described as a waist circumference of more than 40 inches for men and more than 35 inches for women. The patient should be advised to exercise regularly to achieve an ideal body weight. There is no data provided about the patient's BP, so it is not necessary to check it regularly; it is only advisable to eliminate other risk factors. High blood glucose levels increase the risk of developing coronary heart disease but do not help maintain ideal body weight. The patient should be advised not to consume saturated fats because they may lead to additional weight gain and coronary artery disease (CAD).

A patient tells the nurse, "I use nitroglycerin spray when I have pain in my chest, but I don't get any relief." Which questions would the nurse ask to determine a potential cause of the unrelieved pain? Select all that apply. "Are you taking the drug daily?" "How do you feel about taking medication for pain?" "How much are you spraying when you take the drug?" "Do you change positions slowly after taking the medication?" "Are you taking the drug through the nose or under your tongue?

"How much are you spraying when you take the drug?" "Are you taking the drug through the nose or under your tongue?" The patient needs to dose the medication properly to relieve symptoms. The recommended dose is one metered spray on the tongue. The patient should spray the medication on the tongue, rather than inhale it through the nasal route. The medication should be taken as needed. Asking the patient about perceptions about medication-taking will not help determine the cause of unrelieved pain. Taking care to prevent orthostatic hypotension is not related to symptom relief.

The nurse provides medication teaching to a patient who has been prescribed sublingual (SL) nitroglycerin (NTG) tablets. Which statement made by the patient indicates the need for further education? "When I put the tablet under my tongue, I should feel a tingling sensation." "I can continue taking tablets until the pain goes away, each five minutes apart." "I will need to be careful when I stand up because nitroglycerin can cause dizziness." "If chest pain occurs, I will stop what I'm doing and place one tablet under my tongue."

"I can continue taking tablets until the pain goes away, each five minutes apart." The recommended dose for the patient for whom NTG has been prescribed is one tablet taken SL or one metered spray for symptoms of angina. If symptoms are unchanged or worse after five minutes, the patient should contact the emergency medical services (EMS) system before taking additional NTG. Tell the patient to place an NTG tablet under the tongue and allow it to dissolve. NTG should cause a tingling sensation when administered; otherwise it may be outdated. Caution the patient to change positions slowly after NTG use because orthostatic hypotension may occur. The patient experiencing angina should stop the activity and take the NTG.

After teaching a patient with chronic stable angina about nitroglycerin, the nurse recognizes the need for further teaching when the patient makes which statement? "I will replace my nitroglycerin supply every 6 months." "I can take up to 5 tablets every 3 minutes for relief of my chest pain." "I will take acetaminophen (Tylenol) to treat the headache caused by nitroglycerin." "I will take the nitroglycerin 10 minutes before planned activity that usually causes chest pain."

"I can take up to 5 tablets every 3 minutes for relief of my chest pain." Rationale: The recommended dose of nitroglycerin is one tablet taken sublingually (SL) or 1 metered spray for symptoms of angina. If symptoms are unchanged or worse after 5 minutes, the patient should be instructed to activate the emergency medical services (EMS) system. If symptoms are improved, repeat the nitroglycerin every 5 minutes for a maximum of 3 doses and contact EMS if symptoms have not resolved completely.

Which statement made by the student nurse indicates effective learning about the instructions for a patient who is prescribed nitroglycerin (NTG)? "I should direct the patient to inhale the spray containing medication." "I should ask the patient to change position rapidly after the medication use." "I should instruct the patient to report changes in the pattern of pain to the health care provider." "I should encourage the patient to administer the medication every 30 minutes sublingually if symptoms persist."

"I should instruct the patient to report changes in the pattern of pain to the health care provider." Short-acting nitrates such as NTG are first-line medications for angina. The nurse has to instruct the patient on NTG to report any changes in the pattern of pain, frequency of attack, or nocturnal angina to the health care provider. The nurse must direct the patient to spray the medication on the tongue. The nurse should ask the patient to slowly change positions after NTG use to avoid the effects of orthostatic hypotension. The nurse should ask the patient to take NTG every five minutes for a maximum of three doses if symptoms are not resolved.

The nurse is teaching a patient with chronic stable angina about the use of nitroglycerin (NTG). Which statement by the patient indicates effective learning? "I should dissolve the tablets in a glass of water." "I should replace the tablets in the container every 18 months." "I should store the medication in a pocket to have it readily available." "I should take a tablet five minutes before an activity that is known to cause angina

"I should take a tablet five minutes before an activity that is known to cause angina." NTG is a first-line treatment for patients with chronic stable angina. The patient should take NTG 5 to 10 minutes before beginning activities that are known to cause angina pain. The tablets are to be placed under the tongue, not dissolved in water. The patient should keep NTG tablets away from light and heat sources, including body heat, to protect them from degradation. After the container has been opened, the patient should replace the tablets in the container every six months.

The nurse provides information to a patient about ways to decrease risk factors for coronary artery disease (CAD). Which statement by the patient indicates understanding of the teaching? "I will add weightlifting to my daily exercise program." "I will change my diet to increase my intake of saturated fats." "I will change my lifestyle to alter patterns and decrease my stress." "I need to switch to smokeless tobacco instead of smoking cigarettes."

"I will change my lifestyle to alter patterns and decrease my stress." Health-promoting behaviors for those at risk for CAD include altering patterns that are conducive to stress. Other ways to promote health include physical activity, such as brisk walking (three to four miles/hour for at least 30 minutes five or more times a week), reducing total fat and saturated fat intake, and stopping all tobacco use.

The nurse provides discharge teaching to a patient who is newly diagnosed with coronary artery disease (CAD). Which statement made by the patient indicates understanding of the needed dietary modifications? "I will not eat bacon or sausage products." "I will not eat fish more than once a week." "I will eat fried eggs instead of boiled eggs." "I will drink one glass of whole milk per day."

"I will not eat bacon or sausage products." Nutritional guidelines recommended for the patient with CAD include a low-cholesterol and low-fat diet; therefore the patient has to avoid bacon and sausage products. Fish is high in omega-3 fatty acids, which reduce triglyceride levels and can slow the progression of CAD. The American Heart Association (AHA) recommends eating fish twice a week. Egg yolk is high in cholesterol, and the patient with CAD has to avoid fried food. Low-fat or nonfat milk is recommended for the patient with CAD.

The nurse teaches a patient with high cholesterol about natural lipid-lowering therapies. The nurse determines further teaching is necessary when the patient makes which statement? "Omega-3 fatty acids are helpful in reducing triglyceride levels." "I should check with my physician before I start taking any herbal products." "Herbal products do not go through as extensive testing as prescription drugs do." "I will take garlic instead of my prescription medication to reduce my cholesterol."

"I will take garlic instead of my prescription medication to reduce my cholesterol." Rationale: Current evidence does not support using garlic in the treatment of elevated cholesterol. Strong evidence supports the use of omega-3 fatty acids for reduction of triglyceride levels. Many herbal products are not standardized, and effects are not predictable. Patients should consult with their health care provider before starting herbal or natural therapies.

Which statement by an 84-year-old patient with coronary artery disease (CAD) indicates understanding of teaching about physical activity? "I will use longer rest periods between exercise sessions." "I should exercise outside all the time to achieve better results." "I can stop exercising as soon as my cardiac symptoms disappear." "I have to exercise for longer periods and more vigorously than younger people do.

"I will use longer rest periods between exercise sessions." Older adults need longer rest periods between exercise sessions because of decreased endurance and ability to tolerate stress. Older adults have decreased sweating and therefore shouldn't exercise in hot temperatures. Elderly adults have to change their lifestyles to accommodate a physical activity program, even when not experiencing symptoms of CAD. Older adults have to perform low-level activity exercise for longer periods of time.

In caring for the patient with angina, the patient said, "While I was having a bowel movement, I started having the worst chest pain ever, like before I was admitted. I called for a nurse, then the pain went away." What further assessment data should the nurse obtain from the patient? 1. "What precipitated the pain?" 2. "Has the pain changed this time?" 3. "In what areas did you feel this pain?" 4. "What is your pain level on a 0 to 10 scale?"

"In what areas did you feel this pain?" Rationale: Using PQRST, the assessment data not volunteered by the patient is the radiation of pain, the area the patient felt the pain, and if it radiated. The precipitating event was going to the bathroom and having a bowel movement. The quality of the pain was "like before I was admitted," although a more specific description may be helpful. Severity of the pain was the "worst chest pain ever," although an actual number may be needed. Timing is supplied by the patient describing when the pain occurred and that he had previously had this pain.

A patient with angina states, "I walked to the bathroom. While I was having a bowel movement, I started having the worst chest pain ever. It was similar to the pain I had at home. The pain went away rather quickly." Which further subjective assessment data would the nurse obtain? "What precipitated the pain?" "Has the pain changed this time?" "In what location did you feel the pain?" "Please rate the pain on a scale from 0 to 10."

"In what location did you feel the pain?" Using PQRST, the assessment data not volunteered by the patient is the radiation (R) of pain, the area in which the patient felt the pain and if it radiated. The precipitating event (P) was going to the bathroom and having a bowel movement. The quality (Q) of the pain was "like before I was admitted," although a more specific description may be helpful. Severity (S) of the pain was the "worst chest pain ever," although an actual number may be needed. Timing (T) is supplied by the patient describing when the pain occurred and that the patient had had this pain previously.

Which instruction would the nurse give to the patient who is predisposed to coronary artery disease (CAD)? Select all that apply. "Avoid consuming tofu." "Drink whole or 2% milk." "Limit concentrated fruit juice." "Decrease the intake of beans." "Choose foods such as buckwheat and oats."

"Limit concentrated fruit juice." "Choose foods such as buckwheat and oats." Concentrated fruit juices are high in added sugar and should be limited. Buckwheat and oats are made whole grain and are recommended. The American Heart Association (AHA) recommends eating tofu because it contains alpha-linolenic acid, which becomes omega-3 fatty acid in the body. Patients should select fat-free or low-fat dairy products. Beans are a good source of fiber and are recommended.

Which statement made by the student nurse indicates effective learning about cardiovascular disease? "Nicotine stimulates catecholamine release." "Isometric exercise provides relief from angina." "Manifestations of angina often occur in the late evenings." "Decreased heart rate increases myocardial oxygen demand."

"Nicotine stimulates catecholamine release." Chronic stable angina occurs due to insufficient blood flow in the coronary arteries due to atherosclerosis. Nicotine is a precipitating factor that provokes the angina attack because nicotine use stimulates catecholamine release, causing vasoconstriction and increased heart rate. Isometric exercise, such as lifting heavy objects, promotes the onset of angina; the patient should take nitroglycerin 5 to 10 minutes before beginning activities that are known to cause angina. Manifestations of angina occur in the early mornings due to a change in the circadian rhythm patterns. Increased heart rate due to exertion reduces the time the heart spends in diastole, resulting in increased myocardial oxygen demand.

Which instructions would the nurse provide to a patient who takes short-acting nitroglycerin? Select all that apply. "Place the nitroglycerin tablet under the tongue." "Change your position slowly after taking the medicine." "Store nitroglycerin tablets in a warm and bright environment." "Replace nitroglycerin tablets annually once the bottle is opened." "Discard nitroglycerin tablets if they do not cause a tingling sensation.

"Place the nitroglycerin tablet under the tongue." "Change your position slowly after taking the medicine." "Discard nitroglycerin tablets if they do not cause a tingling sensation." Nitroglycerin tablets should be placed under the tongue and allowed to dissolve. Nitroglycerin causes orthostatic hypotension on administration, so the patient should be advised to change position slowly. Nitroglycerin should cause a tingling sensation when administered; if there is not a tingling sensation, the medication should be discarded. The patient should be instructed to store nitroglycerin away from light and heat to protect it from degradation. Once the nitroglycerin bottle is opened, the tablets lose their potency and need to be replaced every six months.

Which advice regarding modifiable risk factors would the nurse give to a patient who has a BP of 150/100 mm Hg? Select all that apply. "Increase total fat intake." "Reduce salt in your diet." "Increase daily physical activity." "Ingest smaller and more frequent meals." "Reduce dietary complex carbohydrates and fiber."

"Reduce salt in your diet." "Increase daily physical activity." BP of 150/100 mm Hg meets the criteria for clinical hypertension. The nurse should advise this patient to consume less salt to lower BP and to exercise daily to reduce the risk of coronary artery disease (CAD) from hypertension. The nurse would advise a reduction in total fat intake for the patient who has elevated serum lipids. In the case of obesity, the nurse would advise the patient to eat smaller and more frequent meals. For the patient who has elevated serum lipids, the nurse would advise an increase the amount of complex carbohydrates, fiber, and vegetable proteins in the diet.

Which statement made by the nurse indicates effective understanding of diagnostic studies used for acute coronary syndrome? "Coronary angiography is not used unless myocardial infarction is suspected." "Serum cardiac markers are proteins that are released from necrotic heart muscle." "A nitroprusside stress echocardiogram is used for patients with acute pericarditis." "A pathogenic Q wave is always present in the electrocardiogram (ECG) of patients with unstable angina."

"Serum cardiac markers are proteins that are released from necrotic heart muscle." Serum cardiac markers such as myoglobin, creatine kinase, cardiac-specific troponin I (cTnI), and cardiac-specific troponin T (cTnT) are released in patients with myocardial infarction (MI) into the blood from necrotic heart muscle. These markers are important to diagnose acute coronary syndrome. Coronary angiography is used for patients with stable or high-risk unstable angina. Pharmacologic stress echocardiogram testing with dobutamine, dipyridamole, or adenosine simulates the effects of exercise and is performed on patients who are unable to exercise or have abnormal, nondiagnostic baseline echocardiograms. A patient with a pathologic Q wave and ST-elevated MI has prolonged coronary occlusion because the MI evolves with time.

The nurse provides information about thrombolytic therapy to a group of student nurses. Which statement made by a student nurse indicates the need for further teaching? "Significant head trauma within the past three months is a relative contraindication." "A symptom of major bleeding with thrombolytic therapy is an increase in heart rate (HR)." "The goal is to give the thrombolytic within 30 minutes of the patient's arrival to the emergency department." "The most reliable sign of blood flow restoration is the return of the ST segment to baseline on the electrocardiogram (ECG)."

"Significant head trauma within the past three months is a relative contraindication." Significant closed-head or facial trauma within the past three months is an absolute contraindication due to increased risk of bleeding; the benefit does not outweigh the risk. Signs and symptoms of bleeding during thrombolytic therapy include a drop in BP, increase in HR, sudden change in the patient's mental status, and blood in the urine or stool. Treatment of ST-segment-elevation myocardial infarction (STEMI) with thrombolytic therapy aims to limit the infarction size by dissolving the thrombus in the coronary artery and reperfusing the heart muscle rapidly. The most reliable sign of blood flow restoration is the return of the ST segment to baseline on the ECG.

A patient phones a health care provider's office and states, "I am having severe chest tightness that won't go away even when I lie down." In addition to contacting emergency responders, which instruction would the nurse provide to the patient? "Lie down with your feet elevated." "Go to a neighbor's house to get assistance." "Take chewable aspirin. The total dose should be 325 mg." "Take your blood pressure. It will be useful to know for your care."

"Take chewable aspirin. The total dose should be 325 mg." The symptoms described by the patient may be due to acute coronary syndrome (ACS). Advise the patient to take chewable aspirin; the dose is 162 to 325 mg (typically 4 baby aspirin or 2 adult aspirin). The patient's BP may be elevated, and lying with the feet elevated will most likely increase the BP. The patient should not increase oxygen demand by walking to a neighbor's house or by taking the BP.

A patient states, "I have episodes of sharp and stabbing chest pain that last 10 minutes when I am restocking shelves at work." The patient is diagnosed with chronic stable angina. Which instruction would the nurse give to the patient to prevent anginal pain while at work? "Perform 30 minutes of moderate exercise before going to work." "Take nitroglycerin (NTG) 5 to 10 minutes before restocking the shelves." "Take a dose of the medication 30 minutes after restocking the shelves." "Take NTG every five minutes during the task until the work is complete."

"Take nitroglycerin (NTG) 5 to 10 minutes before restocking the shelves." Reports of 10 minutes of a sharp, stabbing pain when performing strenuous work indicate that the patient is experiencing an attack of chronic stable angina. The patient should be advised to take NTG prophylactically, 5 to 10 minutes before engaging in the activity. Patients with Prinzmetal's angina may get relief from chest pain by performing moderate exercise. Taking the medication after the activity will not prevent the pain. The maximum number of NTG tablets the patient can take is three.

Which statement made by the student nurse indicates effective learning about Prinzmetal's angina? "The pain may be relieved by moderate exercise." "The patient needs to sleep in recumbent position at night." "The pain is triggered while performing activities of daily living." "The patient should take amphetamines with nitrates during an attack"

"The pain may be relieved by moderate exercise." Prinzmetal's angina occurs due to spasms in the coronary artery even when the patient is at rest. The patient may experience cyclic and short bursts of pain that are relieved upon moderate exercise of the arms. Nocturnal angina occurs when the patient is in recumbent position. Microvascular angina occurs even while performing daily activities due to abnormalities of the coronary microcirculation. The patient should not use nitrates and amphetamines together during an attack.

The nurse teaches a student nurse about acute pericarditis. Which statement made by the student nurse indicates effective learning? "I should massage the patient's chest region to relieve symptoms." "The patient's pain can be relieved by sitting in the forward position." "Acute pericarditis should be treated immediately by IV heparin." "Treatment of acute pericarditis involves mitral valve repair or replacement."

"The patient's pain can be relieved by sitting in the forward position." Acute pericarditis is an inflammation of the pericardium that may occur within two to three days of acute myocardial infarction. The patient's chest pain is relieved after sitting in the forward position. The pain may return after a change in position or inspiration. Massaging the chest region may aggravate the symptoms. IV heparin is given to a patient with an unstable thrombus or coronary artery spasm. Papillary muscle dysfunction treatment involves mitral valve repair or replacement. Acute pericarditis treatment involves nonsteroidal antiinflammatory agents, aspirin, and corticosteroids.

Which statement made by the student nurse indicates effective learning about microvascular angina? "Pain may occur during rapid eye movement (REM) sleep." "Pain occurs at night and is triggered by the patient lying down." "This type of angina is more common in patients with diabetic neuropathy." "This type of angina is more common in women during activities of daily living.

"This type of angina is more common in women during activities of daily living." Microvascular angina is a chest pain associated with abnormalities of the coronary microcirculation, occurring especially in postmenopausal women. Often the angina is prolonged and brought on by physical exertion. With Prinzmetal's angina, pain may occur during REM sleep. Nocturnal angina sets in when the patient lies down and is relieved upon standing or sitting. Silent ischemia occurs in the patient with diabetic neuropathy due to the damage of nerves of the cardiovascular system.

Which instructions would the nurse provide to a patient who receives a prescription for isosorbide mononitrate? Select all that apply. "Tolerance to this long-acting nitroglycerin can develop." "Have your blood pressure measured after the initial dose." "Take the medication only when symptoms of angina appear." "Discontinue the medication if a headache occurs after taking the medication." "You may take acetaminophen along with the medication to relieve headaches."

"Tolerance to this long-acting nitroglycerin can develop." "Have your blood pressure measured after the initial dose." "You may take acetaminophen along with the medication to relieve headaches." Tolerance to the medication may develop; thus patients are scheduled for a 10- to 14-hour nitrate-free period. The patient should have BP measured after the initial dose because isosorbide mononitrate causes venous dilation that may lead to a drop in BP. The patient should be advised to take acetaminophen along with isosorbide mononitrate to avoid a headache due to the dilation of cerebral blood vessels. Isosorbide mononitrate is a long-acting nitrate used to reduce the incidence of anginal attacks and should be taken regularly, not only when angina symptoms are present. The patient should not be advised to discontinue the medication if headaches develop.

A nurse provides discharge teaching to a patient prescribed nitroglycerin sublingual tablets for the treatment of chronic stable angina. Which statement made by the patient indicates the need for further teaching? 1. "I will put the bottle in my pant pocket before I leave the house." 2. "The sublingual tablets expire six months after the bottle is opened." 3. "Possible side effects include a warm feeling, headache, or lightheadedness." 4. "I can use this medication before exercising to prevent angina from occurring."

1. "I will put the bottle in my pant pocket before I leave the house." Sublingual nitroglycerin should not be stored in pant pockets because body heat can cause degradation of the nitroglycerin tablets. Sublingual nitroglycerin tablets tend to lose potency once the bottle has been opened; therefore it should be replaced every six months. Flushing (warm feeling), headache, or dizziness (lightheadedness) may occur following sublingual nitroglycerin administration. Sublingual nitroglycerin can be used prophylactically before starting an activity that is known to cause an anginal attack.

Which instructions would the nurse provide to a patient who takes short-acting nitroglycerin? Select all that apply. 1. "Place the nitroglycerin tablet under your tongue." 2. "Change your position slowly after taking the medicine." 3. "Store nitroglycerin tablets in a warm and bright environment." 4. "Replace nitroglycerin tablets annually once the bottle is opened." 5. "Discard nitroglycerin tablets if they do not cause a tingling sensation."

1. "Place the nitroglycerin tablet under your tongue." 2. "Change your position slowly after taking the medicine." 5. "Discard nitroglycerin tablets if they do not cause a tingling sensation." Nitroglycerin tablets should be placed under the tongue and allowed to dissolve. Nitroglycerin causes orthostatic hypotension on administration, so the patient should be advised to change position slowly. Nitroglycerin should cause a tingling sensation when administered; if there is not a tingling sensation, the medication should be discarded. The patient should be instructed to store nitroglycerin away from light and heat to protect it from degradation. Once the nitroglycerin bottle is opened, the tablets lose their potency and need to be replaced every six months.

In which chronologic order does the progress of atherosclerosis occur? Formation of plaque with collagen over a fatty streak Accumulation of lipids into the smooth muscle cells Proliferation of smooth muscle leading to thickening of arterial wall Adhering of thrombus to the arterial wall leading to complete occlusion

1. Accumulation of lipids into the smooth muscle cells 2. proliferation of smooth muscle leading to thickening of arterial wall 3. Formation of plaque with collagen over a fatty streak 4. Adhering of thrombus to the arterial wall leading to complete occlusion After chronic endothelial injury, the next step in the pathogenesis of atherosclerosis is the accumulation of lipids into smooth muscle cells. Smooth muscle proliferation and thickening of the arterial wall occurs in fibrous plaque stage. Also during this stage, collagen covers the fatty streaks and forms a fibrous plaque with a grayish or whitish appearance. Thrombus, formed by platelet accumulation in large numbers, then adheres to the arterial wall leading to further narrowing or complete occlusion of the artery.

The nurse is caring for a patient who is scheduled for an angioplasty with placement of a drug-eluting stent. In which order do the steps of the stent procedure occur? 1. Balloon is inflated. 2. Blockage is identified. 3. Improved blood flow occurs. 4. Intimal lining grows over the stent. 5. Stent is placed into the coronary artery.

1. Blockage is identified. 2. Stent is placed into the coronary artery. 3. Balloon is inflated. 4. Improved blood flow occurs. 5. Intimal lining grows over the stent. After blockage is identified, treatment options, including angioplasty with stent placement, are explored. Stents are small, expandable tubes that are inserted during angioplasty into a blocked section of the coronary artery to open the artery and improve blood flow. During angioplasty, the balloon is placed inside the stent and inflated, which opens the stent and pushes it into place against the artery wall to keep the narrowed artery open. Because the stent is like woven mesh, the cells lining the blood vessel grow through and around the stent to help secure it. Drug-eluting stents are coated with drugs that prevent scar tissue from growing into the artery. Drug-eluting stents may lower the chance that the patient will need a second procedure (angioplasty or surgery) to open the artery again.

Which drug causes vasodilation by preventing conversion of angiotensin I to angiotensin II? 1. Enalapril 2. Losartan 3. Amlodipine 4. Nitroglycerin

1. Enalapril Enalapril is an angiotensin-converting enzyme (ACE) inhibitor that inhibits the conversion of angiotensin I to angiotensin II, resulting in vasodilation. Losartan is an angiotensin II receptor blocker that inhibits the binding of angiotensin II to angiotensin I receptors. Amlodipine is a calcium channel blocker that prevents calcium entry into the vascular smooth muscles and myocytes. Nitroglycerin is a short-acting nitrate that causes peripheral vasodilation, thereby decreasing preload and afterload.

Which drugs act by causing either direct or indirect vasodilation? 1. Losartan 2. Morphine 3. Reteplase 4. Carvedilol 5. Bivalirudin

1. Losartan 2. Morphine Losartan is an angiotensin II receptor blocker that inhibits binding of angiotensin II to angiotensin I receptors, thereby causing vasodilation. Morphine is an opioid analgesic that acts as a vasodilator to reduce preload and myocardial oxygen consumption. Reteplase is a thrombolytic agent that breaks up the fibrin meshwork in clots. Carvedilol is a beta-adrenergic that reduces the heart rate, contractility, and BP. Bivalirudin is a direct thrombin inhibitor that directly inhibits the clotting factor thrombin.

Cardiac biomarker levels re being evaluated for a patient who is suspected of having a myocardial infarction (MI). Which biomarker lacks specificity for diagnosing an MI? 1. Myoglobin. 2. Creatine kinase-MB (CK-MB) 3. Cardiac-specific troponin I (cTnI) 4. Cardiac-specific troponin II (cTnT)

1. Myoglobin. Myoglobin is serum cardiac marker that is released into the circulation within two hours after an MI. Myoglobin's role in diagnosing MI is limited because it lacks cardiac specificity. Creatine kinase- MB (CK-MB) levels begin to rise about six hours after an MI, are specific to myocardial cells, and help quantify myocardial damage. Cardiac-specific troponin T (cTnT) and cardiac specific troponin I (cTnI) are highly specific indicators of MI.

Place the effects of cigarette smoking on the cardiovascular system in order. Catecholamines are released. Workload on the heart increases. Heart rate and BP increase. Nicotine enters through tobacco smoke.

1. Nicotine enters through tobacco smoke. 3. Catecholamines are released. 4. Heart rate and BP increase. 5. Workload on the heart increases. Nicotine enters the body through the inhalation of tobacco smoke. Nicotine causes the release of the catecholamines epinephrine and norepinephrine. The release of epinephrine and norepinephrine increase the heart rate and the BP. Increased heart rate and BP increase the workload of the heart.

Which therapy would the nurse anticipate administration for emergency care of a patient with a suspected myocardial infarction (MI) ? 1. Oxygen, nitroglycerin, aspirin, and morphine. 2. Aspirin, nitroprusside, dopamine, and oxygen. 3. Nitroglycerine, lorazepam, oxygen, and warfarin. 4. Oxygen, furosemide, nitroglycerine, and meperidine.

1. Oxygen, nitroglycerin, aspirin, and morphine. The American Heart Association's guidelines for emergency care of the patient with chest pain include the administration of oxygen, nitroglycerin, aspirin, and morphine. These interventions serve to relieve chest pain, improve oxygenation, decrease myocardial workload, and prevent further platelet aggregation. Furosemide, meperidine, nitroprusside, dopamine, lorazepam, and warfarin may be used later in the patient's treatment.

Which characteristic is typical of the pain related to chronic stable angina? 1. Relieved by rest, nitroglycerin, or both. 2. Indicates irreversible myocardial damage. 3. Generally lasts longer than 15 to 20 minutes. 4. Frequently associated with vomiting and fatigue.

1. Relieved by rest, nitroglycerin, or both. Chronic stable angina is characterized by intermittent chest pain, often described as pressure or tightness that occurs over a period of time in the same pattern, onset, and intensity. It commonly subsides when precipitating factors have stopped and the patient is at rest or with the use of nitroglycerin. The pain does not always indicate irreversible myocardial damage and usually lasts just 5 to 15 minutes. Vomiting and extreme fatigue are symptoms of myocardial infarction and are not commonly seen in chronic stable angina.

Which factor leads to total occlusion of a coronary artery? 1. Thrombus formation in the arterial wall. 2. Transport of cholesterol into the arterial intima. 3. Smooth muscle proliferation of the arterial wall. 4. Lipid-filled smooth muscle cells in the coronary artery.

1. Thrombus formation in the arterial wall. A thrombus may form and adhere to the wall of the coronary artery, leading to total occlusion. The transport of cholesterol into the arterial intima may lead to the formation of plaques on one portion of the artery, or in a circular fashion involving the entire lumen. Smooth muscle proliferation may lead to the thickening of the arterial wall. The earliest lesions are fatty streaks characterized by lipid-filled smooth muscle cells.

A patient tells the nurse, "I had severe chest pain six days ago." Which cardiac biomarker will be most helpful in determining whether the patient had a myocardial infarction at the time the patient experienced chest pain?" 1. Troponin 2. Myoglobin 3. Homocysteine 4. Creatine Kinase (CK)

1. Troponin Troponin is a serum cardiac marker that is detectable in the blood up to two weeks after myocardial injury and is used to diagnose a myocardial infarction. Troponin has two subtypes: cardiac-specific troponin T (cTnT) and cardiac-specific troponin I (cTnI). Serum levels of cTnI increases 4 to 6 hours after the onset of myocardial injury, peak at 10 to 24 hours, and returns to baseline over 10 to 14 days. Myoglobin is a protein found in skeletal and cardiac muscle. It is a sensitive indicator of early myocardial injury but is not specific for cardiac muscle; therefore it is not used to diagnose myocardial infarction. Myoglobin peaks and returns to normal 3 to 15 hours. Homocysteine is a protein. High levels of homocysteine may indicate an increased risk for coronary artery disease. It is not used to diagnose myocardial infarction. CK levels begin to rise about 6 hours after an MI, peak at about 18 hours, and return to normal within 24 to 36 hours.

Which type of angina increases in frequency, duration, and severity over time? 1. Unstable angina. 2. Prinzmetal's angina. 3. Microvascular angina. 4. Chronic stable angina.

1. Unstable angina. Unstable angina increases in frequency, and severity as the time progresses. Prinzmetal's angina occurs primarily at rest and is triggered by smoking and increased levels of substances, such as histamine and epinephrine. Microvascular angina is triggered by activity of daily life and exertion. Chronic stable angina is provoked by exertion and relieved by stress.

Tips Related to Food Choices and Preparation

1. Use the Nutrition Facts panel on food labels and ingredients list when choosing foods to buy. 2. Use fresh or frozen vegetables and fruits in place of canned vegetables and fruits. 3. Replace high-calorie foods with fresh fruits and vegetables. 4. Increase fiber intake by eating beans (legumes), whole-grain products, fruits, and vegetables. 5. Use liquid vegetable oils in place of solid fats. 6. Limit beverages and foods high in added sugars (e.g., sucrose, glucose, fructose, maltose, dextrose, corn syrups, concentrated fruit juice, honey). 7. Choose foods made with whole grains (e.g., whole wheat, oats, rye, barley, brown rice, wild rice, buckwheat). 8. Avoid pastries and high-calorie bakery products (e.g., muffins, doughnuts). 9. Select milk and dairy products that are either fat free or low fat. 10. Reduce salt intake by • Comparing the sodium content of similar products (e.g., different brands of tomato sauce) and choosing products with less sodium. • Choosing versions of processed foods with reduced salt, including cereals, canned products, and baked goods. • Limiting condiments (e.g., soy sauce, ketchup). 11. Use lean cuts of meat and remove skin from poultry before cooking or eating. 12. Avoid processed meats that are high in saturated fat and sodium (e.g., deli meats). 13. Grill, bake, or broil fish, meat, and poultry. 14. Incorporate plant-based meat substitutes into recipes (e.g., soy, tofu, quinoa). 15. Consume whole vegetables and fruits in place of juices.

Which drug produces an anticoagulant effect by interfering with hepatic synthesis of vitamin K-dependent clotting factors? 1. Warfarin 2. Prasugrel 3. Argatroban 4. Eptifibatide

1. Warfarin Warfarin interferes with hepatic synthesis of the vitamin K-dependent clotting factors that result in an anticoagulant effect. Prasugrel causes an antiplatelet effect by inhibiting platelet aggregation. Argatroban acts by directly inhibiting the clotting factor thrombin and results in an anticoagulant effect. Eptifibatide prevents the binding of fibrinogen to platelets, thereby blocking platelet aggregation.

The community health nurse is planning health promotion teaching to prevent coronary artery disease (CAD). Which population has the highest incidence of CAD? 1. White male. 2. Hispanic male. 3. Black male. 4. Native American female

1. White male. The incidence of CAD and myocardial infarction (MI) is highest among white, middle-aged men. Hispanic individuals have lower rates of CAD than non-Hispanic whites or blacks. Blacks have an earlier age of onset and more severe CAD than whites and more than twice the mortality rate of whites of the same age. Native Americans have increased mortality in 35-year-old-and -under people and have major modifiable risk factors, such as diabetes.

Which assessment data is a nonmodifiable risk factor for coronary artery disease? 1. 72-year-old white male. 2. 20-pack-a-year former cigarette smoker. 3. Adult onset diabetes, hyperlipidemia. 4. Abdominal aortic aneurysm repair.

1. white male. White race and male sex are nonmodifiable factors for coronary artery disease that the patient cannot change. Diabetes is modifiable through diet, medication, and exercise for tight blood sugar control. Hyperlipidemia is modifiable through medication, diet, and exercise to control cholesterol and triglyceride levels. Cigarette smoking is modifiable through the cessation of tobacco use.

Which drug breaks up the fibrin meshwork in blood clots? 1. Nadolol 2. Alteplase 3. Valsartan 4. Nicardipine

2. Alteplase Alteplase is a thrombolytic agent that breaks up the fibrin meshwork in clots. Nadolol is a beta-adrenergic blocker that inhibits sympathetic nervous stimulation of the heart. Valsartan is an angiotensin II receptor blocker, which inhibits the binding of angiotensin II to angiotensin I receptors. Nicardipine is a calcium channel blocker that prevents calcium entry into the vascular smooth muscles and cardiac cells.

Which drug class includes clopidogrel? 1. Opioid analgesic. 2. Antiplatelet agent. 3. Vitamin K antagonist. 4. Nonsteroidal anti-inflammatory drug (NSAID)

2. Antiplatelet agent. Clopidogrel (Plavix) is an antiplatelet agent used commonly in patients with cardiovascular disease. Plavix is not an opioid analgesic, vitamin K antagonist, or NSAID.

The nurse provides information to a patient about preventing coronary artery disease (CAD) by maintaining healthy serum low-density lipoprotein (LDL) and high-density lipoprotein (HDL) levels. Which goals would the nurse emphasize? 1. Decreased LDLs; decreased HDLs 2. Decreased LDLs; increased HDLs 3. Increased LDLs; increased HDLs 4. Increased LDLs; decreased HDs

2. Decreased LDLs; increased HDLs. LDLs contain more cholesterol than any other lipoprotein and have an attraction to arterial walls, whereas HDLs carry lipids away from the arteries to the liver for metabolism and prevent lipid accumulation within the arterial walls. Therefore increasing HDL levels and decreasing LDL levels are most helpful in lowering the patient's risk for coronary artery disease. The nurse should not advise the patient to decrease HDLs or increase LDLs because these actions would be counterproductive.

The nurse provides a student nurse with information related to management of a patient following cardiac catheterization. Which intervention would the nurse explain is the specific responsibility of the nurse, rather than that of assistive personnel (AP)? 1. Recording vital signs. 2. Monitoring neurovascular changes. 3. Asking the patient about comfort measures. 4. Assisting with oral hygiene, hydration, and meals.

2. Monitoring neurovascular changes. The nurse is responsible for monitoring changes in neurovascular status or bleeding. AP take vital signs. All members of the health care team can address measures that will make the patient more comfortable. AP assist with oral hygiene, hydration, meals and toileting.

The nurse assesses a patient who has been diagnosed with papillary muscle dysfunction. Which observation supports the patient's diagnosis? 1. S3 heart sound. 2. Murmur at the cardiac apex. 3. Crackles in the bilateral lung bases. 4. Deep sound at the lower left sternal border.

2. Murmur at the cardiac apex. Papillary muscle dysfunction occurs if the papillary muscles attached to the mitral valves are involved in infarction. A patient may have papillary muscle infarction if a murmur is heard upon auscultation at the cardiac apex. The presence of S3 and S4 sounds of the heart and crackles in breath sounds indicate heart failure. The deep sound heard at the lower left sternal border upon auscultation indicates acute pericarditis.

For which reason would a patient be prescribed low-molecular-weight heparin (LMWH) after a stent placement? 1. To stabilize the stent and prevent dislodgement. 2. To prevent thrombosis formation within the stent. 3. To prevent an anaphylactic reaction related to the stent anaphylaxis. 4. To allow for smooth surface of the intimal lining that grows over the stent.

2. To prevent thrombosis formation within the stent. LMWH is taken to prevent thrombosis formation within the stent. LMWH does not stabilize the stent or prevent displacement or prevent anaphylaxis. LMWH is used short term during the percutaneous coronary intervention. Aspirin (indefinitely) and ticagrelor or clopidogrel are used for months, until intimal lining grows over the stent and provides a smooth vascular surface.

The nurse is caring for a patient who survived sudden cardiac death (SCD) caused by a lethal ventricular dysrhythmia. Which tests will likely be prescribed to monitor the effectiveness of drug treatment? Select all that apply. 24-hour Holter monitoring Implantable cardiac monitor MRI Signal-averaged electrocardiogram (ECG) Electrophysiologic study (EPS) under fluoroscopy

24-hour Holter monitoring Implantable cardiac monitor Signal-averaged electrocardiogram (ECG) Electrophysiologic study (EPS) under fluoroscopy Because most SCD patients have lethal ventricular dysrhythmias associated with a high recurrence rate, they are closely monitored to assess when they are most likely to have a recurrence and to determine which drug therapies are most effective for them. This monitoring may include 24-hour Holter monitoring, implantable cardiac monitor, signal-averaged ECG, and an EPS done under fluoroscopy. MRI is not used to monitor for lethal dysrhythmias.

Which is a correct term for the development of arterial anastomoses? 1. Atheromas. 2. Angioedema. 3. Angiogenesis. 4. Atherosclerosis.

3. Angiogenesis. Angiogenesis (development of new blood vessels) best describes the development of arterial anastomoses, which are new arterial connections forming between two vessels that are not usually connected. Atheromas (atheromatous plaques) are fatty deposits that form within and on the layers of the coronary artery walls during atherosclerosis. Atherosclerosis is an arterial disease characterized by gradual arterial narrowing due to the atheroma formation within and on the coronary artery walls. If arterial blockage from atherosclerosis forms very gradually, the process of angiogenesis has been known to create collateral circulation around the arterial blockage. Angioedema is the rapid severe swelling of the soft tissues ( e.g., skin or mucosa) caused by medications, allergies, or an inherited condition.

In which stage of arterial disease does the plaque rupture in the artery? 1. Fatty streak. 2. Fibrous plaque. 3. Complicated lesion. 4. Chronic endothelial injury.

3. Complicated lesion. The rupture of plaque in the artery occurs when the lesion is complicated. The plaque ruptures, and thrombus formation occurs, as does further narrowing or total occlusion of the vessel. Fibrous plaque occurs when collagen covers the fatty streak, narrowing the artery and thus reducing the blood flow. When the fatty streak forms, lipids accumulate and migrate into the smooth muscle cells. Chronic endothelial injury occurs secondary to exposure of toxins, infections, and certain disease processes.

Which drug binds to antithrombin III, enhancing its effects? 1. Warfarin. 2. Bivalirudin. 3. Dalteparin. 4. Ticagrelor.

3. Dalteparin binds to antithrombin III, enhancing its effects. Warfarin interferes with hepatic synthesis of vitamin K-dependent clotting factors, producing an anticoagulant effect. Bivalirudin acts by directly inhibiting the clotting factor, thrombin. Ticargrelor inhibits platelet aggregation and is used along with aspirin.

Which drug reduces triglycerides by decreasing the hepatic synthesis and secretion of very-low-density lipoproteins (VLDLs)? 1. Niacin 2. Fluvastatin 3. Gemfibrozil 4. Cholestyramine

3. Gemfibrozil Gemfibrozil decreases the hepatic synthesis and secretion of VLDLs, which reduces the levels of triglycerides. Niacin inhibits the synthesis of VLDLs and low-density lipoproteins (LDLs). Fluvastatin blocks the synthesis of cholesterol and increases the LDL receptors in the liver. Cholestyramine binds with the bile acids in the intestines, forming an insoluble complex. The binding results in the removal of LDL and cholesterol.

Which drug acts by direct inhibition of the clotting factor thrombin? 1. Heparin 2. Felodipine 3. Enoxaparin 4. Argatroban

4. Argatroban Argatroban is a direct thrombin inhibitor, which directly inhibits the clotting factor thrombin. Heparin is an anticoagulant, which acts by preventing conversion of fibrinogen to fibrin and prothrombin to thrombin. Felodipine is a calcium channel blocker, which prevents the entry of calcium into vascular smooth muscles and myocytes. Enoxaparin is a low-molecular-weight-heparin, which binds to antithrombin III and enhances its effects.

The nurse provides a list of health-promoting regular physical activity examples to a patient with coronary artery disease (CAD). Which activity would the nurse include on the list? 1. Painting while seated. 2. Performing carpentry. 3. Jogging (7 to 8 miles per hour). 4. Brisk walking (3 to 4 miles her hour).

4. Brisk walking (3 to 4 miles per hour). Physical activity improves the physiologic functioning and psychological well-being of a patient with acute coronary syndrome. Therefore the nurse should encourage the patient to walk at a rate of 3 to 4 miles per hour. Painting while seated is a low-energy activity. Carpentry and running at 7 miles per hour are high-energy activities for a patient with acute coronary syndrome.

The nurse is caring for a patient 24 hours after the patient after the patient was diagnosed with ST-segment-elevation myocardial infarction (STEMI). For which complication of myocardial infarction (MI) would the nurse monitor this patient? 1. Unstable angina. 2. Cardiac tamponade. 3. Sudden cardiac death. 4. Cardiac dysrhythmias.

4. Cardiac dysrhythmias. The most common complications after MI is dysrhythmias, which are present in 80% of patients. Unstable angina is considered a precursor to MI rather than a complication. Cardiac tamponade is a rare event, and sudden cardiac death is defined as an unexplained death from cardiac causes. Cardiac dysfunction in the period following an MI would not be characterized as sudden cardiac death.

A patient who survived an episode of sudden cardiac death (SCD) is recovering in the intensive care unit (ICU). Which intervention would the nurse anticipate to prevent a recurrence? 1. Drug therapy with beta-blocker. 2. Coronary artery bypass graft (CABG). 3. Percutaneous coronary intervention (PCI). 4. Implantable cardioverter-defibrillator (ICD).

4. Implantable cardioverter-defibrillator. The most common approach to preventing a recurrence is the use of an ICD. It has been shown that an ICD improves survival compared with drug therapy alone. Drug therapy and a PCI will not prevent a recurrence of SCD.

A patient who has received a maximum dose of nitroglycerin continues to report chest pain. Which medication would the nurse anticipate administering to this patient next? 1. Esmolol. 2. Docusate. 3. Ticagrelor. 4. Morphine sulfate.

4. Morphine sulfate. Morphine sulfate is the drug of choice for a patient with unrelieved chest pain, even after the administration of nitroglycerin. Esmolol is a beta-blocker used to slow down the heart during minimal invasive direct coronary artery bypass (MIDCAB). Docusate is a stool softener that facilitates bowel movements. Ticagrelor is used in dual antiplatelet therapy on a patient with ongoing angina and negative cardiac markers.

Which finding associated with myocardial ischemia could be obtained by a cardiac catheterization? Cardiac enlargement ST segment depression Abnormal cardiac wall motion 70% block in right coronary artery

70% block in right coronary artery Cardiac catheterization is an invasive diagnostic procedure to find out the location and severity of blockages in the coronary circulation. ST segment depression is an important diagnostic finding for the presence of myocardial ischemia, which is obtained by electrocardiography (ECG). Cardiac enlargement is a sign of heart failure that can be seen on an x-ray. Echocardiography is used to detect the presence of abnormal wall motion due to myocardial ischemia.

Which person would the nurse identify as having the highest risk for coronary artery disease (CAD)? 1. A 60-yr-old man with low homocysteine levels 2. A 45-yr-old man with a high-stress job who is depressed 3. A 54-yr-old woman vegetarian with increased high-density lipoprotein (HDL) levels 4. A 62-yr-old woman who has a sedentary lifestyle and body mass index (BMI) of 23 kg/m2

A 45-yr-old man with a high-stress job who is depressed Rationale: The 45-yr-old depressed man with a high-stress job is at the highest risk for CAD. Depression and stressful states can contribute to the development of CAD. Elevated HDL levels and low homocysteine levels help to prevent CAD. Although a sedentary lifestyle is a risk factor, a BMI of 23 kg/m2 depicts normal weight, and thus the patient with two risk factors is at greatest risk for developing CAD.

Which patient is at greatest risk for sudden cardiac death (SCD)? A 52-yr-old black man with left ventricular failure A 62-yr-old obese man with diabetes and high cholesterol A 42-yr-old white woman with hypertension and dyslipidemia A 72-yr-old Native American woman with a family history of heart disease

A 52-yr-old black man with left ventricular failure Rationale: Patients with left ventricular dysfunction (ejection fraction less than 30%) and ventricular dysrhythmias after myocardial infarction are at greatest risk for SCD. Other risk factors for SCD include: (1) male gender (especially blacks), (2) family history of premature atherosclerosis, (3) tobacco use, (4) diabetes, (5) high cholesterol levels, (6) hypertension, and (7) cardiomyopathy.

A patient had a non-ST-segment-elevation myocardial infarction (NSTEMI) 3 days ago. Which nursing intervention is appropriate for the registered nurse (RN) to delegate to an experienced licensed practical/vocational nurse (LPN/VN)? a. Reinforcement of teaching about the prescribed medications b. Evaluation of the patient's response to walking in the hallway c. Completion of the referral form for a home health nurse follow-up d. Education of the patient about the pathophysiology of heart disease

ANS: A LPN/VN education and scope of practice include reinforcing education that has previously been done by the RN. Evaluating the patient's response to exercise after a NSTEMI requires more education and should be done by the RN. Teaching and discharge planning and referral are skills that require RN education and scope of practice.

A patient who has recently started taking pravastatin (Pravachol) and niacin reports several symptoms to the nurse. Which information is most important to communicate to the health care provider? a. Generalized muscle aches and pains b. Dizziness with rapid position changes c. Nausea when taking the drugs before meals d. Flushing and pruritus after taking the drugs

ANS: A Muscle aches and pains may indicate myopathy and rhabdomyolysis, which have caused acute kidney injury and death in some patients who have taken the statin medications. These symptoms indicate that the pravastatin may need to be discontinued. The other symptoms are common side effects when taking niacin, and although the nurse should follow-up with the health care provider, they do not indicate that a change in medication is needed.

A patient who is being admitted to the emergency department with intermittent chest pain gives the following list of daily medications to the nurse. Which medication has the most immediate implications for the patient's care? a. Tadalafil (Cialas) b. Furosemide (Lasix) c. Warfarin (Coumadin) d. Diltiazem (Cardizem)

ANS: A The nurse will need to avoid giving nitrates to the patient because nitrate administration is contraindicated in patients who are using tadalafil because of the risk of severe hypotension caused by vasodilation. The other home medications should be documented and reported to the health care provider but do not have as immediate an impact on decisions about the patient's treatment.

In preparation for discharge, the nurse teaches a patient with chronic stable angina how to use the prescribed short-acting and long-acting nitrates. Which patient statement indicates that the teaching has been effective? a. "I will sit down before I put the nitroglycerin under my tongue." b. "I will check my pulse rate before I take any nitroglycerin tablets." c. "I will put the nitroglycerin patch on as soon as I get any chest pain." d. "I will remove the nitroglycerin patch before taking sublingual nitroglycerin."

ANS: A The patient should sit down before taking the nitroglycerin to decrease cardiac workload and prevent orthostatic hypotension. Transdermal nitrates are used prophylactically rather than to treat acute pain and can be used concurrently with sublingual nitroglycerin. Although the nurse should check blood pressure before giving nitroglycerin, patients do not need to check the pulse rate before taking nitrates.

After having a myocardial infarction (MI) and successful percutaneous coronary intervention, the patient states, "It was just a little chest pain. As soon as I get out of here, I'm going for my vacation as planned." Which reply would be most appropriate for the nurse to make? a. "What do you think caused your chest pain?" b. "Where are you planning to go for your vacation?" c. "Sometimes plans need to change after a heart attack." d. "Recovery from a heart attack takes at least a few weeks."

ANS: A When the patient is experiencing denial, the nurse should assist the patient in testing reality until the patient has progressed beyond this step of the emotional adjustment to MI. Asking the patient about vacation plans reinforces the patient's plan, which is not appropriate in the immediate post-MI period. Reminding the patient in denial about the MI is likely to make the patient angry and lead to distrust of the nursing staff.

A patient who has recently had an acute myocardial infarction (AMI) ambulates in the hospital hallway. Which data would indicate to the nurse that the patient should stop and rest? a. O2 saturation drops from 99% to 95%. b. Heart rate increases from 66 to 98 beats/min. c. Respiratory rate goes from 14 to 20 breaths/min. d. Blood pressure (BP) changes from 118/60 to 126/68 mm Hg.

ANS: B A change in heart rate of more than 20 beats over the resting heart rate indicates that the patient should stop and rest. The increases in BP and respiratory rate, and the slight decrease in O2 saturation, are normal responses to exercise.

The nurse is caring for a patient who has just arrived on the telemetry unit after having cardiac catheterization. What task should the nurse delegate to a licensed practical/vocational nurse (LPN/VN)? a. Teach the patient about the postprocedure plan of care. b. Give the scheduled aspirin and lipid-lowering medication. c. Perform the initial assessment of the catheter insertion site. d. Titrate the heparin infusion according to the agency protocol.

ANS: B Administration of oral medications is within the scope of practice for LPNs/VNs. The initial assessment of the patient, patient teaching, and titration of IV anticoagulant medications should be done by the registered nurse (RN).

To improve the physical activity level for a mildly obese 68-year-old patient, which action should the nurse plan to take? a. Stress that weight loss is a major benefit of increased exercise. b. Determine what kind of physical activities the patient usually enjoys. c. Tell the patient that older adults should exercise for no more than 20 minutes at a time. d. Teach the patient to include a short warm-up period at the beginning of physical activity.

ANS: B Because patients are more likely to continue physical activities that they already enjoy, the nurse will plan to ask the patient about preferred activities. The goal for older adults is 30 minutes of moderate activity on most days. Older adults should plan for a longer warm-up period. Benefits of exercises, such as improved activity tolerance, should be emphasized rather than aiming for significant weight loss in older mildly obese adults.

The nurse is admitting a patient who has chest pain. Which assessment data suggest that the pain is from an acute myocardial infarction? a. The pain increases with deep breathing. b. The pain has lasted longer than 30 minutes. c. The pain is relieved after the patient takes nitroglycerin. d. The pain is reproducible when the patient raises the arms.

ANS: B Chest pain that lasts for 20 minutes or more is characteristic of AMI. Changes in pain that occur with raising the arms or with deep breathing are more typical of musculoskeletal pain or pericarditis. Stable angina is usually relieved when the patient takes nitroglycerin.

Which patient statement indicates that the nurse's teaching about carvedilol (Coreg) for preventing anginal episodes has been effective? a. "Carvedilol will help my heart muscle work harder." b. "It is important not to suddenly stop taking the carvedilol." c. "I can expect to feel short of breath when taking carvedilol." d. "Carvedilol will increase the blood flow to my heart muscle."

ANS: B Patients who have been taking beta-adrenergic blockers can develop intense and frequent angina if the medication is suddenly discontinued. Carvedilol (Coreg) decreases myocardial contractility. Shortness of breath that occurs when taking beta-adrenergic blockers for angina may be due to bronchospasm and should be reported to the health care provider. Carvedilol works by decreasing myocardial O2 demand, not by increasing blood flow to the coronary arteries.

A patient who is recovering from an acute myocardial infarction (AMI) asks the nurse about safely resuming sexual intercourse. Which response by the nurse is best? a. "Most patients are able to enjoy intercourse without any complications." b. "Sexual activity uses about as much energy as climbing two flights of stairs." c. "The doctor will provide sexual guidelines when your heart is strong enough." d. "Holding and cuddling are good ways to maintain intimacy after a heart attack."

ANS: B Sexual activity places about as much physical stress on the cardiovascular system as moderate-energy activities, such as climbing two flights of stairs. The other responses are general statements that may be accurate, but do not provide useful guidelines for judging the physical safety of the activity.

Which assessment finding in a patient who has had coronary artery bypass grafting using a right radial artery graft is most important for the nurse to communicate to the health care provider? a. Complaints of incisional chest pain b. Pallor and weakness of the right hand c. Fine crackles heard at both lung bases d. Redness on both sides of the sternal incision

ANS: B The changes in the right hand indicate compromised blood flow, which requires immediate evaluation and actions, such as prescribed calcium channel blockers or surgery. The other changes are expected or require nursing interventions.

Which action will the nurse take to evaluate the effectiveness of IV nitroglycerin for a patient with a myocardial infarction (MI)? a. Monitor heart rate. b. Ask about chest pain. c. Check blood pressure. d. Observe for dysrhythmias.

ANS: B The goal of IV nitroglycerin administration in MI is relief of chest pain by improving the balance between myocardial oxygen supply and demand. The nurse will also monitor heart rate and blood pressure and observe for dysrhythmias, but these parameters will not indicate whether the medication is effective.

Which patient at the cardiovascular clinic requires the most immediate action by the nurse? a. Patient with type 2 diabetes whose current blood glucose level is 145 mg/dL. b. Patient with stable angina whose chest pain has recently increased in frequency. c. Patient with familial hypercholesterolemia and a total cholesterol of 465 mg/dL. d. Patient with chronic hypertension whose blood pressure today is 172/98 mm Hg.

ANS: B The history of more frequent chest pain suggests that the patient may have unstable angina, which is part of the acute coronary syndrome spectrum. This will require rapid implementation of actions such as cardiac catheterization and possible percutaneous coronary intervention. The data about the other patients suggest that their conditions are more stable.

The nurse is evaluating the effectiveness of preoperative teaching with a patient scheduled for coronary artery bypass graft (CABG) surgery using the internal mammary artery. Which patient statement indicates that additional teaching is needed? a. "They will circulate my blood with a machine during surgery." b. "I will have incisions in my leg where they will remove the vein." c. "They will use an artery near my heart to go around the area that is blocked." d. "I will need to take aspirin every day after the surgery to keep the graft open."

ANS: B When the internal mammary artery is used, there is no need to have a saphenous vein removed from the leg. The other statements by the patient are accurate and indicate that the teaching has been effective.

Which nursing intervention is likely to be most effective when assisting the patient with coronary artery disease to make appropriate dietary changes? a. Inform the patient about a diet containing no saturated fat and minimal salt. b. Emphasize the increased cardiac risk unless the patient makes dietary changes. c. Help the patient modify favorite high-fat recipes by using monounsaturated oils. d. Give the patient a list of low-sodium, low-cholesterol foods to include in the diet.

ANS: C Lifestyle changes are more likely to be successful when consideration is given to the patient's values and preferences. The highest percentage of calories from fat should come from monounsaturated or polyunsaturated fats. Although low-sodium and low-cholesterol foods are appropriate, providing the patient with a list alone is not likely to be successful in making dietary changes. Completely removing saturated fat from the diet is not a realistic expectation. Up to 7% of calories in the therapeutic lifestyle changes diet can come from saturated fat. Telling the patient about the increased risk without assisting further with strategies for dietary change is unlikely to be successful.

Which data indicates to the nurse that the patient with stable angina is experiencing a side effect of metoprolol (Lopressor)? a. Patient is restless and agitated. b. Patient reports feeling anxious. c. Blood pressure is 90/54 mm Hg. d. Heart monitor shows normal sinus rhythm.

ANS: C Patients taking beta-adrenergic blockers should be monitored for hypotension and bradycardia. Because this class of medication inhibits the sympathetic nervous system, restlessness, agitation, hypertension, and anxiety will not be side effects. Normal sinus rhythm is a normal and expected heart rhythm.

Diltiazem (Cardizem) is prescribed for a patient with newly diagnosed Prinzmetal's (variant) angina. Which action of diltiazem is accurate for the nurse to include in the teaching plan? a. Reduces heart palpitations. b. Prevents coronary artery plaque. c. Decreases coronary artery spasms. d. Increases contractile force of the heart.

ANS: C Prinzmetal's angina is caused by coronary artery spasm. Calcium channel blockers (e.g., diltiazem, amlodipine [Norvasc]) are a first-line therapy for this type of angina. Lipid-lowering drugs help reduce atherosclerosis (i.e., plaque formation), and beta-adrenergic blockers decrease sympathetic stimulation of the heart (i.e., palpitations). Medications or activities that increase myocardial contractility will increase the incidence of angina by increasing O2 demand.

A patient with hyperlipidemia has a new order for colesevelam (Welchol). Which nursing action is appropriate when scheduling this medication? a. Administer the medication at the patient's usual bedtime. b. Have the patient take the colesevelam 1 hour before breakfast. c. Give the patient's other medications 2 hours after colesevelam. d. Have the patient take the dose at the same time as the prescribed aspirin.

ANS: C The bile acid sequestrants interfere with the absorption of many other drugs and giving other medications at the same time should be avoided. Taking an aspirin concurrently with the colesevelam may increase the incidence of gastrointestinal side effects such as heartburn. For maximum effect, colesevelam should be administered with meals.

The nurse is administering a thrombolytic agent to a patient with an acute myocardial infarction. What patient data indicates that the nurse should stop the drug infusion? a. Bleeding from the gums b. An increase in blood pressure c. Decreased level of consciousness d. A nonsustained episode of ventricular tachycardia

ANS: C The change in level of consciousness indicates that the patient may be experiencing intracranial bleeding, a possible complication of thrombolytic therapy. Some bleeding of the gums is an expected side effect of the therapy but not an indication to stop infusion of the thrombolytic medication. A decrease in blood pressure could indicate internal bleeding. A nonsustained episode of ventricular tachycardia is a common reperfusion dysrhythmia and may indicate that the therapy is effective.

The nurse obtains the following data when assessing a patient who experienced an ST-segment-elevation myocardial infarction (STEMI) 2 days previously. Which information is most important to report to the health care provider? a. The troponin level is elevated. b. The patient denies having a heart attack. c. Bilateral crackles in the mid-lower lobes. d. Occasional premature atrial contractions (PACs).

ANS: C The crackles indicate that the patient may be developing heart failure, a possible complication of myocardial infarction (MI). The health care provider may need to order medications such as diuretics or angiotensin-converting enzyme inhibitors for the patient. Elevation in troponin level at this time is expected. PACs are not life-threatening dysrhythmias. Denial is a common response in the immediate period after the MI.

The nurse reviews information shown in the accompanying figure from the medical records of a 43-year-old patient. Which risk factor modification for coronary artery disease should the nurse include in patient teaching? a. Importance of daily physical activity b. Effect of weight loss on blood pressure c. Dietary changes to improve lipid levels d. Cardiac risk associated with previous tobacco use

ANS: C The patient has an elevated low-density lipoprotein cholesterol and low high-density lipoprotein cholesterol, which will increase the risk of coronary artery disease. The patient's waist circumference and body mass index indicate an appropriate body weight. The risk for coronary artery disease a year after quitting smoking is the same as a nonsmoker. The patient's occupation indicates that daily activity is at the levels suggested by national guidelines.

Which electrocardiographic (ECG) change by a patient with chest pain is most important for the nurse to report rapidly to the health care provider? a. Inverted P wave b. Sinus tachycardia c. ST-segment elevation d. First-degree atrioventricular block

ANS: C The patient is likely to be experiencing an ST-segment-elevation myocardial infarction. Immediate therapy with percutaneous coronary intervention or thrombolytic medication is indicated to minimize myocardial damage. The other ECG changes may also suggest a need for therapy but not as rapidly.

A patient with acute coronary syndrome has returned to the coronary care unit after having angioplasty with stent placement. Which assessment data indicate the need for immediate action by the nurse? a. Report of chest pain b. Heart rate 102 beats/min c. Pedal pulses 1+ bilaterally d. Blood pressure 103/54 mm Hg

ANS: C The patient's chest pain indicates that restenosis of the coronary artery may be occurring and requires immediate actions, such as administration of oxygen and nitroglycerin, by the nurse. The other information indicates a need for ongoing assessments by the nurse.

A patient recovering from a myocardial infarction (MI) develops chest pain on day 3 that increases when taking a deep breath and is relieved by leaning forward. Which action should the nurse take as focused follow-up on this symptom? a. Assess both feet for pedal edema. b. Palpate the radial pulses bilaterally. c. Auscultate for a pericardial friction rub. d. Check the heart monitor for dysrhythmias.

ANS: C The patient's symptoms are consistent with the development of pericarditis, a possible complication of MI. The other assessments listed are not consistent with the description of the patient's symptoms.

A patient who has chest pain is admitted to the emergency department (ED), and all of the following items are prescribed. Which one should the nurse arrange to be completed first? a. Chest x-ray b. Troponin level c. Electrocardiogram (ECG) d. Insertion of a peripheral IV

ANS: C The priority for the patient is to determine whether an acute myocardial infarction (AMI) is occurring so that the appropriate therapy can begin as quickly as possible. ECG changes occur very rapidly after coronary artery occlusion, and an ECG should be obtained as soon as possible. Troponin levels will increase after about 3 hours. Data from the chest x-ray may impact the patient's care but are not helpful in determining whether the patient is experiencing a myocardial infarction. Peripheral access will be needed but not before the ECG.

A patient with diabetes mellitus and chronic stable angina has a new order for captopril. What should the nurse teach this patient about the primary purpose of captopril? a. Decreases the heart rate. b. Controls blood glucose levels. c. Prevents changes in heart muscle. d. Reduces the frequency of chest pain.

ANS: C The purpose for angiotensin-converting enzyme (ACE) inhibitors in patients with chronic stable angina who are at high risk for a cardiac event is to decrease ventricular remodeling. ACE inhibitors do not directly impact angina frequency, blood glucose, or heart rate.

A patient with ST-segment elevation in three contiguous electrocardiographic leads is admitted to the emergency department and diagnosed as having an ST-segment-elevation myocardial infarction (STEMI). Which question should the nurse ask to determine whether the patient is a candidate for thrombolytic therapy? a. "Do you have any allergies?" b. "Do you take aspirin daily?" c. "What time did your pain begin?" d. "Can you rate the pain on a 0 to 10 scale?"

ANS: C Thrombolytic therapy should be started within 6 hours of the onset of the myocardial infarction, so the time at which the chest pain started is a major determinant of the appropriateness of this treatment. The other information is not a factor in the decision about thrombolytic therapy.

Which statement made by a patient with coronary artery disease after the nurse has completed teaching about the therapeutic lifestyle changes (TLC) diet indicates that further teaching is needed? a. "I will switch from whole milk to 1% milk." b. "I like salmon and I will plan to eat it more often." c. "I can have a glass of wine with dinner if I want one." d. "I will miss being able to eat peanut butter sandwiches."

ANS: D Although only 30% of the daily calories should come from fats, most of the fat in the TLC diet should come from monounsaturated fats such as are found in nuts, olive oil, and canola oil. The patient can include peanut butter sandwiches as part of the TLC diet. The other patient comments indicate a good understanding of the TLC diet.

Nadolol (Corgard) is prescribed for a patient with chronic stable angina and left ventricular dysfunction. What data would indicate to the nurse that the drug is effective? a. Decreased blood pressure and heart rate b. Improvement in the strength of the distal pulses c. Fewer complaints of having cold hands and feet d. Participation in daily activities without chest pain

ANS: D Because the drug is ordered to improve the patient's angina, effectiveness is indicated if the patient is able to accomplish daily activities without chest pain. Blood pressure and heart rate may decrease, but these data do not indicate that the goal of decreased angina has been met. The noncardioselective beta-adrenergic blockers can cause peripheral vasoconstriction, so the nurse would not expect an improvement in distal pulse quality or skin temperature.

A patient admitted to the coronary care unit (CCU) with an ST-segment-elevation myocardial infarction (STEMI) is restless and anxious. The blood pressure is 86/40 mm Hg, and heart rate is 132 beats/min. Based on this information, which patient problem is the priority? a. Anxiety b. Acute pain c. Stress management d. Decreased cardiac output

ANS: D The hypotension and tachycardia indicate decreased cardiac output and shock from the damaged myocardium. This will result in decreased perfusion to all vital organs (e.g., brain, kidney, heart) and is a priority.

A patient who has had chest pain for several hours is admitted with a diagnosis of rule out acute myocardial infarction (AMI). Which laboratory test is most specific for the nurse to monitor in determining whether the patient has had an AMI? a. Myoglobin b. Homocysteine c. C-reactive protein d. Cardiac-specific troponin

ANS: D Troponin levels increase about 4 to 6 hours after the onset of myocardial infarction (MI) and are highly specific indicators for MI. Myoglobin is released within 2 hours of MI, but it lacks specificity and its use is limited. The other laboratory data are useful in determining the patient's risk for developing coronary artery disease but are not helpful in determining whether an acute MI is in progress.

The patient is being dismissed from the hospital after acute coronary syndrome (ACS) and will be attending rehabilitation. What information would be taught in the early recovery phase of rehabilitation? 1. Therapeutic lifestyle changes should become lifelong habits. 2. Physical activity is always started in the hospital and continued at home. 3. Attention will focus on managing chest pain, anxiety, dysrhythmias, and other complications. 4. Activity level is gradually increased under cardiac rehabilitation team supervision and monitoring.

Activity level is gradually increased under cardiac rehabilitation team supervision and monitoring. Rationale: In the early recovery phase after the patient is dismissed from the hospital, the activity level is gradually increased under supervision and with ECG monitoring. The late recovery phase includes therapeutic lifestyle changes that become lifelong habits. In the first phase of recovery, activity is dependent on the severity of the angina or myocardial infarction, and attention is focused on the management of chest pain, anxiety, dysrhythmias, and other complications. With early recovery phase, the cardiac rehabilitation team may suggest that physical activity be initiated at home, but this is not always done.

A patient who is being discharged from the hospital after acute coronary syndrome will be participating in cardiac rehabilitation. Which information will the nurse provide about the early recovery phase of rehabilitation? Activity level depends on severity of angina or myocardial infarction (MI). Therapeutic lifestyle changes should become lifelong habits. Activity level is increased gradually with supervision and with electrocardiogram (ECG) monitoring. The focus will be on management of chest pain, anxiety, dysrhythmias, and other complications.

Activity level is increased gradually with supervision and with electrocardiogram (ECG) monitoring. In the early recovery phase after the patient is dismissed from the hospital, the activity level is increased gradually under supervision and with ECG monitoring. In the first phase of recovery, activity is dependent on the severity of the angina or MI. The late recovery phase includes therapeutic lifestyle changes that become lifelong habits. In the first phase of recovery attention is focused on the management of chest pain, anxiety, dysrhythmias, and other complications.

A patient experiences prolonged chest pain that is not immediately reversible. The patient's health care provider explains that the cause of the pain is that a once-stable atherosclerotic plaque has ruptured, causing platelet aggregation and thrombus formation. Which condition is consistent with this explanation? Unstable angina Acute coronary syndrome (ACS) ST-segment-elevation myocardial infarction (STEMI) Non-ST-segment-elevation myocardial infarction (NSTEMI)

Acute coronary syndrome (ACS) When ischemia is prolonged and not immediately reversible, ACS develops. ACS is associated with deterioration of a once-stable atherosclerotic plaque that ruptures, exposes the intima to blood, and stimulates platelet aggregation and local vasoconstriction with thrombus formation. The unstable lesion, if partially occlusive, will manifest as unstable angina or NSTEMI. If there is total occlusion, it is manifest as STEMI.

For which problem is percutaneous coronary intervention (PCI) most clearly indicated? 1. Chronic stable angina 2 Left-sided heart failure 3. Coronary artery disease 4. Acute myocardial infarction

Acute myocardial infarction Rationale: PCI is indicated to restore coronary perfusion in cases of myocardial infarction. Chronic stable angina and coronary artery disease are normally treated with more conservative measures initially. PCI is not relevant to the pathophysiology of heart failure.

A nurse is preparing a patient for cardiac catheterization and percutaneous coronary intervention. In addition to vital signs, pulse oximetry, and heart and lung sounds, which other vital assessment would the nurse include? Anemia Allergies Dysrhythmia Mental status

Allergies Before performing a cardiac catheterization, the nurse should assess the patient for an allergy to contrast medium, which would have an immediate adverse effect on the patient receiving this procedure. Anemia, dysrhythmia, and change in mental status present less immediate complications during a cardiac catheterization procedure.

Which type of medication may be prescribed for a patient with an ejection fraction (EF) of 25%? Lipid-lowering agent β-adrenergic blocker Calcium channel blocker Angiotensin-converting enzyme (ACE) inhibitor

Angiotensin-converting enzyme (ACE) inhibitor Patients with chronic stable angina who have an EF of 40% or less should take an ACE inhibitor indefinitely, unless contraindicated. These drugs result in vasodilation and reduced blood volume. They also prevent ventricular remodeling and prevent the progression of heart failure in the patient. Lipid-lowering agents reduce low density cholesterol and triglyceride levels in the blood. β-adrenergic blockers decrease myocardial oxygen demand by reducing heart rate, BP, and contractility. Calcium channel blockers are prescribed to decrease BP in patients with β-blocker intolerance and Prinzmetal's angina.

A patient reports heaviness and burning sensation in the substernal and retrosternal region. Which assessment finding would indicate sympathetic nervous system stimulation? Jugular venous distention Abnormal S3 and S4 sounds Ashen, clammy, and cool skin Shortness of breath and anxiety

Ashen, clammy, and cool skin A patient with heaviness and burning sensation in the substernal and retrosternal region may be having a myocardial infarction (MI) and stimulation of the sympathetic nervous system. This condition may trigger the production of catecholamines, which promote glycogen release, diaphoresis, and vasoconstriction of peripheral blood vessels. Because of this, the patient's skin may appear ashen, clammy, and cool to the touch. Jugular vein distention and abnormal S3 and S4 sounds are caused by ventricular dysfunction. Shortness of breath and anxiety may not necessarily be caused by sympathetic nervous system stimulation.

Postoperative care of a patient undergoing coronary artery bypass graft (CABG) surgery includes monitoring for which common complication? 1. Dehydration 2. Paralytic ileus 3. Atrial dysrhythmias 4. Acute respiratory distress syndrome

Atrial dysrhythmias Rationale: Postoperative dysrhythmias, specifically atrial dysrhythmias, are common in the first 3 days after CABG surgery. Although the other complications could occur, they are not common complications.

A patient who has stable angina is prescribed a short-acting nitrate medication. Which assessment parameter would the nurse plan to monitor for effects of this medication? Heart rate (HR) QT interval BP Ejection fraction

BP BP is the most important patient assessment parameter for the nurse to monitor in a patient taking short-acting nitrates (i.e., nitroglycerin) due to the possibility of experiencing orthostatic hypotension. This patient should be monitored regularly and instructed to rise from the bed slowly and carefully. HR and QT interval would not be expected to change after nitrates. Ejection fraction is not routinely monitored and is measured during diagnostic studies.

Complimentary & alternative lipid-lowering methods

Berberine -reduces total cholesterol, low-density lipoproteins (LDLs), triglycerides Garlic -Conflicting evidence. -reduces total cholesterol and LDL over short periods Green tea -reduces total cholesterol Flaxseed -Conflicting evidence -reduces total cholesterol Omega-3 fatty acids (fish oil) -reduces cholesterol and triglyceride levels Black psyllium -reduces total cholesterol and LDL Plant sterols -reduces total cholesterol and LDL Red yeast rice -reduces total cholesterol, LDL, and triglycerides. - contains a chemical similar to statins Soy -slight reduction in total cholesterol and LDL

Complimentary & alternative lipid lowering agents

Berberine -reducing total cholesterol, low-density lipoproteins (LDLs), triglycerides Garlic -Conflicting evidence. -reducing total cholesterol and LDL over short periods Green tea -reducing total cholesterol Flaxseed -reducing total cholesterol Omega-3 fatty acids (fish oil) -reducing cholesterol and triglyceride levels Black psyllium -reducing total cholesterol and LDL Plant sterols -reducing total cholesterol and LDL Red yeast rice -reducing total cholesterol, LDL, and triglycerides. -contains a chemical similar to statins Soy -slight reduction in total cholesterol and LDL

Drugs That Increase Lipoprotein Removal

Bile-acid sequestrants: -increase conversion of cholesterol to bile acids in the liver and decrease hepatic cholesterol -primary effect is a decrease in total cholesterol and LDL levels Side effects: -taste -upper and lower GI symptoms -belching -heartburn -nausea -abdominal pain -constipation. Decrease absorption of other drugs: -warfarin -thiazides -thyroid hormones -β-adrenergic blockers [β-blockers] -digoxin -thiazide diuretics -warfarin -some antibiotics [e.g., penicillins]) -Separating time between medications by at least 2 hours decreases this adverse effect.

A patient receives morphine sulfate to relieve chest pain. For which adverse effects would the nurse monitor the patient? Select all that apply. Bradypnea Dysrhythmias Bradycardia Hypotension Decreased ejection fraction

Bradypnea Hypotension Morphine sulfate is prescribed to patients with chest pain. Morphine acts as a vasodilator; it decreases cardiac workload, contractility, and BP. The nurse should monitor for signs of bradypnea and hypotension to avoid myocardial ischemia and infarction. The nurse should monitor dysrhythmias and bradycardia in a patient receiving docusate. Morphine sulfate does not directly affect ejection fraction; indirectly, improved oxygenation will lead to more effective cardiac pumping.

The nurse recognizes additional teaching is needed when the patient prescribed a low-sodium, low-fat cardiac diet selects which food? 1. Baked flounder 2. Angel food cake 3. Canned chicken noodle soup 4. Baked potato with margarine

Canned chicken noodle soup Rationale: Canned soups are very high in sodium content. Patients need to be taught to read food labels for sodium and fat content.

A patient with a history of diabetes is diagnosed with chronic stable angina. Which drug that decreases endothelial dysfunction would the nurse expect to teach to this patient? Diltiazem Sirolimus Captopril Bivalirudin

Captopril A patient with a history of diabetes who has chronic stable angina has a high risk for a cardiac event. An angiotensin-converting enzyme (ACE) inhibitor, such as captopril, can be safely used in this patient; it decreases endothelial dysfunction and prevents conversion of angiotensin I to angiotensin II, resulting in vasodilation. Calcium channel blockers, such as diltiazem, mask the signs of hypoglycemia and are used cautiously in patients with diabetes. Sirolimus is used in drug-eluting stents that prevent the overgrowth of new intima, the primary cause of stent restenosis. Bivalirudin helps prevent the abrupt closure of the stents during percutaneous coronary intervention.

Which drug may prevent or limit ventricular remodeling? Captopril Valsartan Metoprolol Nitroglycerin

Captopril Captopril is an angiotensin-converting enzyme (ACE) inhibitor that may prevent or limit ventricular remodeling. Valsartan is an angiotensin II receptor blocker that is used in patients who are intolerant to angiotensin-converting enzyme (ACE) inhibitor. Metoprolol is a β-adrenergic blocker that reduces heart rate, contractility, and blood pressure. Nitroglycerin is a nitrate that promotes coronary artery vasodilation.

The nurse auscultates a new murmur at the cardiac apex in a patient who has had a myocardial infarction. Which treatment strategies would the nurse expect may be included in the patient's collaborative care plan? Select all that apply. Antiplatelet agents Short-term corticosteroids Cardiac surgery with mitral valve repair Intraaortic balloon pump (IABP) therapy Nonsteroidal antiinflammatory drugs (NSAIDs)

Cardiac surgery with mitral valve repair Intraaortic balloon pump (IABP) therapy Papillary muscle dysfunction is a complication of myocardial infarction and should be suspected if a new murmur at the cardiac apex is detected. It may occur if the infarcted area includes or is near the papillary muscle that attaches the mitral valve. The patient should be treated with nitroprusside, IABP therapy, or cardiac surgery with mitral valve repair to reduce the afterload of the heart. Antiplatelets, such as aspirin and NSAIDs, help treat acute pericarditis. Short-acting corticosteroids are effective in the treatment of Dressler syndrome.

The registered nurse observes a student nurse who is administering transdermal controlled-release nitroglycerin (NTG) to a patient. Which action performed by the student nurse indicates the need for further teaching? Wears gloves to apply the medication Changes the transdermal patch every 24 hours Applies the medication on an area that is flat and muscular Informs the patient that headaches are common with this medication

Changes the transdermal patch every 24 hours NTG helps reduce the chest pain in the patient with angina. It acts by decreasing cardiac workload. The preparation should be removed in the evening to allow for a 10- to 14-hour nitrate-free interval. The nurse should wear gloves while applying the medication to avoid contact with the ointment, which may cause systemic absorption of the drug. Headaches are common after taking any NTG preparation. The NTG ointment should be placed on an area with no hair or scars that is flat and muscular.

A patient who takes digoxin daily is being treated for hyperlipidemia. Which prescription would the nurse question? Niacin Icosapent Atorvastatin Cholestyramine

Cholestyramine Fibric acid derivatives, such as cholestyramine, interfere with the absorption of many drugs including digoxin. Icosapent ethyl is an omega-3 fatty acid. Niacin is in the broad category name of niacin. Atorvastatin is an HMG-CaA reductase inhibitor (statin).

Place the formation of coronary artery disease in the order in which it develops. Fibrous plaque Chronic endothelial injury Fatty streak Complicated lesion

Chronic endothelial injury Fatty streak Fibrous plaque Complicated lesion Coronary artery disease development begins with chronic endothelial injury from health issues like hypertension, diabetes, and tobacco use. A fatty streak develops with the accumulation of lipids. A fibrous plaque develops over the fatty streak. Disruption of the fatty streak leads to formation of a complicated lesion.

A patient who has angina is being seen in a health care provider's office. While in the office, the patient experiences chest pain and takes a total of three nitroglycerin tablets, five minutes apart. The patient's chest pain is not relieved. Which is the nurse's priority action? Assess a complete set of vital signs. Contact the emergency medical services (EMS). Repeat the medication in another five minutes. Instruct the patient to lie quietly and take deep breaths.

Contact the emergency medical services (EMS) A patient with angina who takes sublingual nitroglycerin and whose condition worsens or does not resolve after administration of the medication should be referred to an EMS immediately for help. The drug should be administered to the patient every five minutes for a maximum of three doses. The nurse should take measures to reduce the severity of the condition first and then monitor vital signs and provide patient instructions.

A patient taking atorvastatin reports a recent onset of muscle weakness and pain. Which laboratory studies will the nurse anticipate scheduling? Serum electrolytes Renal function tests Thyroid function tests Creatine kinase isoenzymes

Creatine kinase isoenzymes The patient with weak muscles (myopathy) and breakdown of skeletal muscles (rhabdomyolysis) is showing the adverse effects of treatment with HMG-CoA reductase inhibitors, such as atorvastatin. Creatine kinase isoenzymes (e.g., CK-MM) should be assessed if symptoms of myopathy occur. Performing electrolytes, thyroid function tests, and/or renal function tests are not necessary for the patient described.

Which high-density lipoprotein (HDL) and low-density lipoprotein (LDL) results would lead the nurse to plan teaching to reduce a patient's risk of coronary artery disease (CAD)? Increased HDLs; increased LDLs Decreased HDLs; decreased LDLs Increased HDLs; decreased LDLs Decreased HDLs; increased LDLs

Decreased HDLs; increased LDLs The risk of CAD is associated with increased LDLs (>160 mg/dL) and decreased HDLs (< 40 mg/dL).

Which other assessment finding would the nurse expect in a patient who reports substernal pain radiating to the neck that occurs for a few minutes when exercising? Decrease in the heart rate Depression of the ST segment Persistent pain even when resting Frequent episodes of migraine headache

Depression of the ST segment Stable angina pain relates to substernal pain lasting 5 to 15 minutes radiating to the neck and jaw. The 12-lead electrocardiogram (ECG) in a patient with angina has a depression in the ST segment and may also have T wave inversion. The patient with physical exertion has an increased heart rate, which reduces the time the heart spends in diastole, resulting in an increase in myocardial oxygen demand. The pain due to stable angina is usually relieved when the exercise stops. Migraine headache is seen patients taking nitroglycerin due to dilation of the cerebral blood vessels.

Which change occurs first that makes patients susceptible to coronary artery disease? Formation of thrombus in the coronary artery Formation of fibrous plaque in the endothelium Reduction of blood flow through the coronary arteries Development of fatty streaks in the smooth muscles of the endothelium

Development of fatty streaks in the smooth muscles of the endothelium The first stage of coronary artery disease is the development of fatty streaks within the smooth muscles. The fibrous plaque development increases with age. As age progresses, the fibrous plaque becomes larger in size. This compromises the integrity of the inner arterial wall. As more platelets get accumulated, there is thrombus formation. There is a reduction of blood flow through the coronary arteries because they are blocked due to the formation of fibrous plaque.

Which drugs act by decreasing the contractility of the heart? Select all that apply. Captopril Diltiazem Valsartan Carvedilol Morphine

Diltiazem Carvedilol Diltiazem is a calcium channel blocker that reduces the heart rate, contractility, and BP. Carvedilol is a β-adrenergic blocker that also reduces the heart rate, contractility, and BP. Captopril is an angiotensin-converting enzyme (ACE) inhibitor that causes vasodilation by preventing conversion of angiotensin I to angiotensin II. Valsartan is an angiotensin II receptor blocker that causes vasodilation by inhibiting the binding of angiotensin II to angiotensin I receptors. Morphine is an opioid analgesic that acts as a vasodilator and reduces preload and myocardial oxygen consumption.

Which intervention would the nurse plan for a patient who smokes cigarettes and is predisposed to developing coronary artery disease? Recommend smokeless tobacco. Encourage the use of filtered cigarettes. Suggest smoking low-nicotine cigarettes. Discuss medication to help with tobacco cessation.

Discuss medication to help with tobacco cessation. The patient must be encouraged to quit smoking. Smoking cessation medications, such as bupropion, can be used to prevent the withdrawal symptoms of nicotine. Even if the patient is using smokeless tobacco, the risk of developing coronary artery disease is the same. Changing to filtered cigarettes does not affect the risk of developing coronary artery disease. Changing from high-nicotine to low-nicotine cigarettes also does not affect the risk of getting coronary heart disease.

The nurse is caring for a patient one month after the patient had a myocardial infarction. The patient is hospitalized with a three-day history of chest pain, joint pain, and a body temperature of 101° F (38.3° C). The patient's lab results include a white blood cell count of 15,000/mcL and an erythrocyte sedimentation rate of 30 mm/hr. Which condition would the nurse suspect? Pneumonia Hiatal hernia Dressler syndrome Ventricular aneurysm

Dressler syndrome Dressler syndrome is pericarditis that develops four to six weeks after myocardial infarction. This syndrome is caused by an antigen-antibody reaction to the necrotic myocardium, and the patient may experience pericardial pain, fever, and arthralgia. Laboratory findings of an elevated white blood cell count and sedimentation rate also indicate Dressler syndrome. Note that the normal level of white blood cells is 10,000/cc and the normal range of sedimentation rate is 0 to 22 mm/hr for men and 0 to 29 mm/hr for women. Pneumonia and hiatal hernia can cause chest pain that requires emergency management. Ventricular aneurysm results from thinning of myocardial wall during contraction.

A patient admitted to the emergency department 24 hours ago with reports of chest pain was diagnosed with a ST-segment-elevation myocardial infarction (STEMI). What complication of myocardial infarction should the nurse anticipate? 1. Dysrhythmias 2. Unstable angina 3. Cardiac tamponade 4. Sudden cardiac death

Dysrhythmias Rationale: Dysrhythmias are present in 80% to 90% of patients after myocardial infarction (MI). Unstable angina is considered a precursor to MI rather than a complication. Cardiac tamponade is a rare event, and sudden cardiac death is defined as an unexpected death from cardiac causes. Cardiac dysfunction in the period following an MI would not be characterized as sudden cardiac death.

A nurse teaches a patient about strategies to prevent angina caused by coronary artery disease. The nurse tells the patient to rest for one to two hours after a heavy meal. What is the rationale for this instruction? Eating a heavy meal would divert more blood to the gastrointestinal system. Heavy meals cause obesity and increase the susceptibility to myocardial ischemia. Eating a heavy meal can cause physical inactivity, which could precipitate angina. Heavy meals cause excessive heat production, which leads to peripheral vasodilation.

Eating a heavy meal would divert more blood to the gastrointestinal system. The digestive system requires more blood supply for a longer period of time to digest heavy meals. Therefore blood is diverted to the gastrointestinal system, which causes reduced blood supply to the myocardium. Eating heavy meals causes obesity in due course of time, and this increases the susceptibility of an individual to have coronary artery disease. Physical inactivity does not cause an anginal attack; angina can be precipitated by physical exertion. Eating heavy meals does not cause peripheral pooling of blood.

The nurse is caring for a patient with liver impairment. Which prescription would the nurse question? Niacin Ezetimibe Icosapent ethyl Cholestyramin

Ezetimibe Ezetimibe should not be used by patients with liver impairment. Niacin's side effects subside with time, although decreased liver function may occur with high doses. Icosapent ethyl has no major liver-related side effects. Cholestyramine is safe for long-term use.

For which patient conditions is coronary revascularization with coronary artery bypass graft (CABG) surgery recommended? Select all that apply. Severe aortic stenosis Failed medical management Left main coronary artery or three-vessel disease Not candidates for percutaneous coronary intervention (PCI) Failed PCI with continued chest pain

Failed medical management Left main coronary artery or three-vessel disease Not candidates for percutaneous coronary intervention (PCI) Failed PCI with continued chest pain Coronary revascularization with CABG is recommended for patients who have diabetes mellitus, have left main coronary artery or three-vessel disease, patients who are not candidates for percutaneous intervention and who have failed percutaneous intervention and continue to have chest pain, and those who are expected to have longer-term benefits with this surgery than with catheterization. Severe aortic stenosis is not an indicator for cardiac revascularization surgery.

Which assessment finding should be considered when caring for a woman with suspected coronary artery disease? 1. Fatigue may be the first symptom. 2. Classic signs and symptoms are expected. 3. Increased risk is present before menopause. 4. Women are more likely to develop collateral circulation.

Fatigue may be the first symptom. Rationale: Fatigue, rather than pain or shortness of breath, may be the first symptom of impaired cardiac circulation. Women may not exhibit the classic signs and symptoms of ischemia such as chest pain which radiates down the left arm. Neck, throat, or back pain may be symptoms experienced by women. Risk for coronary artery disease increases four times after menopause. Men are more likely to develop collateral circulation.

Which symptoms in a 55-year-old female patient would the nurse recognize as possibly indicating unstable angina? Dyspnea, hyperglycemia, and polyuria Altered mentation with difficulty breathing Fatigue, indigestion, and shortness of breath Peripheral edema with decreased urinary output

Fatigue, indigestion, and shortness of breath An adult female with fatigue, indigestion, and dyspnea may be experiencing an unstable anginal (UA) attack. A patient experiencing dyspnea, hyperglycemia, and polyuria may have diabetes. Altered mentation (confusion) and difficulty breathing could be caused by many conditions, including heart failure exacerbation, sepsis, or UA in an older patient. Peripheral edema with decreased urine output may indicate right heart failure due to a history of UA or coronary artery disease.

Gemfibrozil (Lopid)

Fenofibric acid derivative -aid in the removal of VLDLs -lowering triglycerides and increasing HDL levels -have no effect on LDLs -GI irritability is common -caution when combined with statin medications due to increased risk for myopathy. • May increase the risk for bleeding in patients taking warfarin (Coumadin) • Increases the risk of hypoglycemia in patients taking repaglinide (Prandin)

A nurse assesses a patient whose angiogram revealed an 80% blockage of the left circumflex artery and 70% blockage of the right coronary artery. The patient does not show any symptoms of coronary ischemia. Which reason explains this finding? Production of C-reactive proteins Formation of collateral circulation Pulmonary artery supplies oxygenated blood to the heart Lowering of low-density lipoprotein (LDL) levels in the body

Formation of collateral circulation Collateral circulation develops as an inherited predisposition to develop new blood vessels or in the presence of chronic ischemia. With sufficient collateral circulation, the heart may still receive an adequate amount of blood and oxygen; therefore the patient is asymptomatic. The pulmonary artery consists of deoxygenated blood and does not supply blood to the coronary arteries unless there is a congenital anatomic variation. C-reactive proteins are inflammatory markers that are increased in patients with coronary artery disease. These are not associated with reducing coronary ischemia. Lowering LDL levels does not prevent coronary ischemia in the patient whose myocardial blood supply is already compromised.

A patient's lipid profile indicates hyperlipidemia. Which rationale supports the nurse advising the patient to consume a diet to increase high-density lipoprotein (HDL) levels? Select all that apply. HDLs prevent stiffening of arterial walls. HDLs transport lipids to the liver for metabolism. HDLs stimulate the liver to break down more low-density lipoproteins (LDLs). HDLs prevent deposition of lipids in the blood vessels. HDLs inhibit the production of LDLs.

HDLs transport lipids to the liver for metabolism. HDLs prevent deposition of lipids in the blood vessels. HDLs mobilize lipids from the arteries to the liver for metabolism, thereby preventing the deposition of lipids on the vessel wall. HDLs do not directly alter the arterial wall or make it noncompliant. HDLs do not interfere with the production or breakdown of LDLs.

A 78-year-old patient has a diagnosis of hypertension with a cholesterol level of 250 mg/dL and a fasting triglyceride level of 195 mg/dL. Which level of risk does this patient have for coronary artery disease (CAD)? None Low High Unable to determine

High Increasing age is a nonmodifiable risk factor for coronary artery disease. Hypertension is the second major risk factor in CAD. The risk of CAD is associated with a serum cholesterol level greater than 200 mg/dL or a fasting triglyceride level greater than 150 mg/dL. This patient has several risk factors for CAD and is therefore considered high risk. The patient has too many risk factors to be considered low or at no risk, and additional information is not necessary to determine high risk.

A patient experienced sudden cardiac death (SCD) and survived. Which treatment should the nurse expect to be implemented to prevent an SCD recurrence at home? 1. External cardiac pacemaker 2. An electrophysiologic study (EPS) 3. Medications to prevent dysrhythmias 4. Implantable cardioverter-defibrillator (ICD)

Implantable cardioverter-defibrillator (ICD) Rationale: An ICD is the most common approach to preventing recurrence of SCD. An external pacemaker may be used in the hospital but will not be used for the patient living daily life at home. An EPS may be done to determine if a recurrence is likely and determine the most effective medication treatment. Medications to prevent dysrhythmias are used but are not the best prevention of SCD.

Which information will the nurse include when educating an older adult patient about physical activity to reduce the risk for coronary artery disease? Select all that apply. Include longer warm-up periods. Plan indoor exercise on hot summer days. Include shorter periods of low-level activity. Include shorter rest periods between sessions. 5 Exercise a minimum of 60 minutes on most days.

Include longer warm-up periods. Plan indoor exercise on hot summer days. Longer warm-up periods are recommended for older adult patients to prevent injury and additional stress. Because older adults have a decreased ability to sweat efficiently, they are at a higher risk for heat intolerance and should therefore be encouraged to plan exercise indoors when it is hot outside. An older adult's physical activity program should include longer, not shorter, rest periods between sessions, longer, not shorter, periods of low-level activity, and exercising at a minimum of 30 minutes, not 60 minutes, on most days of the week as able.

In addition to troponin levels, which laboratory result in a patient's health record is most indicative of myocardial infarction? Increased myoglobin Increased C-reactive protein Increased creatine kinase-MB (CK-MB) Increased white blood cell count

Increased creatine kinase-MB (CK-MB) Biochemical markers, such as creatine kinase (CK) and troponin are released specifically by myocardial cells when injured and are detectable in the blood. The CK enzymes are fractionated into bands. The CK-MB is specific to heart muscles and helps to quantify myocardial damage. Myoglobin, although one of the first markers to increase after a myocardial infarction (MI), does not have as high of a cardiac specificity as others. C-reactive protein is increased after an MI as a result of the inflammation caused by tissue damage; however, it is also not as highly specific to cardiac tissue. An increased white blood cell count may be present after an MI but is due to a generalized inflammatory response.

A patient with angina is prescribed a calcium channel blocker. Upon reviewing the medication history, the nurse finds that the patient is on digoxin. For which complication would the nurse monitor? Increase in weight Prolonged QT interval Decrease in BP Increased serum digoxin level

Increased serum digoxin level The nurse should closely monitor the serum digoxin levels for toxicity because calcium channel blockers potentiate the action of digoxin by increasing serum digoxin levels. Weight should be monitored in patients taking β-blockers. QT wave prolongation should be monitored upon administration of sodium current inhibitors. Long-acting nitrates, such as isosorbide dinitrate and isosorbide mononitrate, can cause hypotension, resulting in orthostatic hypotension.

In which location would a myocardial infarction (MI) occur due to blockage of the right coronary artery? Inferior wall Anteroseptal Anterolateral Anterior wall

Inferior wall Blockage of the right coronary artery may result in an inferior wall MI because the right coronary artery supplies blood to the inferior wall of the heart. Damage to one or more other coronary arteries may result in anteroseptal and anterolateral MIs. Anterior wall infarctions result from blockages in the left anterior descending artery

The nurse in the recovery room assesses the right femoral artery puncture site after the patient had a stent inserted into a coronary artery. The insertion site is not bleeding or discolored. What should the nurse do next to ensure the femoral artery is intact? Palpate the insertion site for induration. Assess peripheral pulses in the right leg. Inspect the patient's right side and back. Compare the color of the left and right legs.

Inspect the patient's right side and back. Rationale: The best method to determine that the right femoral artery is intact after inspection of the insertion site is to logroll the patient to inspect the right side and back for retroperitoneal bleeding. The artery can be leaking, and blood is drawn into the tissues by gravity. The peripheral pulses, color, and sensation of the right leg will be assessed per agency protocol.

The nurse is reviewing a plan of care for emergency treatment of a patient with chest pain. Which item listed on the plan would the nurse question? Give a high-dose statin medication. Give 162 to 325 mg aspirin (chewable). Start O2 by nasal cannula to keep O2 saturation above 93%. Instruct the patient to do coughing and deep-breathing exercises.

Instruct the patient to do coughing and deep-breathing exercises. Rapid diagnosis and providing treatment to a patient with acute coronary syndrome help to preserve cardiac muscle function. The initial treatment is to manage chest pain; therefore the patient needs to rest and limit activities (including breathing exercises) for 12 to 24 hours. The nurse should make sure that the oxygen saturation stays at an acceptable level by initiating supplemental oxygen. Aspirin is part of the antiplatelet therapy. Statins are lipid-lowering drugs. They block synthesis of cholesterol and increase low-density lipoprotein receptors in the liver.

Which artery is most commonly used for coronary bypass grafts? Radial Gastroepiploic Inferior epigastric Internal mammary

Internal mammary Bypass graft surgery involves the replacement of blood vessels that transport blood between the aorta and the blocked coronary artery. The internal mammary artery (IMA) is the most common artery used for bypass graft. The long-term patency rate for an IMA graft is greater than 90% after 10 years. Procedures involving the radial, gastroepiploic, or inferior epigastric artery have a comparatively short-term patency rate.

For which reason would a patient with acute coronary syndrome (ACS) be scheduled for an off-pump coronary artery bypass (OPCAB) surgery rather than a traditional surgical approach? It does not involve a sternotomy. It is associated with decreased postoperative spasms. It is associated with less blood loss and less renal dysfunction. It uses a robot to perform the surgery, which allows for increased precision

It is associated with less blood loss and less renal dysfunction. The OPCAB is a procedure that allows for access to all coronary vessels. OPCAB is useful for patients with multiple comorbidities because it is associated with less blood loss, less renal dysfunction, less postoperative atrial fibrillation, and fewer neurologic complications. It involves a median sternotomy. This type of surgery does not have an advantage of decreased postoperative spasms. The robotic or totally endoscopic coronary artery bypass (TECAB) surgery uses a robot to perform coronary artery bypass grafting (CABG) surgery.

A female patient with type 1 diabetes has chronic stable angina controlled with rest. She states that over the past few months, she has required increasing amounts of insulin. What goal should the nurse use in planning care to prevent cardiovascular disease progression? Exercise almost every day. Avoid saturated fat intake. Limit calories to daily limit. Keep Hgb A1C less than 7%.

Keep Hgb A1C less than 7%. Rationale: If the Hgb A1C is kept below 7%, this means that the patient has had good control of her blood glucose over the past 3 months. The patient indicates that increasing amounts of insulin are being required to control her blood glucose. This patient may not be adhering to the dietary guidelines or therapeutic regimen, so teaching about how to maintain diet, exercise, and medications to maintain stable blood glucose levels will be needed to achieve this goal.

Modifiable Risk Factors

Major Serum lipids: • Total cholesterol >200 mg/dL • Triglycerides ≥150 mg/dL∗ • LDL cholesterol >130 mg/dL • HDL cholesterol <40 mg/dL in men or <50 mg/dL in women∗ • BP >120/80 mm Hg∗ -Diabetes∗ • Tobacco use • Physical inactivity • Obesity: Waist circumference ≥102 cm (≥40 in) in men and ≥88 cm (≥35 in) in women∗ Contributing • Psychosocial risk factors (e.g., depression, hostility • High homocysteine levels • Substance abuse

Which condition is likely in the female patient who reports chest pain while performing daily activities and has no significant coronary atherosclerosis? Silent ischemia Nocturnal angina Angina decubitus Microvascular angina

Microvascular angina The patient may have microvascular angina, which is chest pain that occurs in the absence of significant coronary atherosclerosis or coronary spasm; it is especially common in women. The chest pain is related to myocardial ischemia associated with abnormalities of coronary microcirculation. If the ischemia occurs in the absence of any subjective symptoms, then the condition is silent ischemia. Nocturnal angina occurs only at night. Angina decubitus occurs only when the patient is lying down or is in a recumbent position and is relieved when the patient is in the sitting or standing position.

The nurse is caring for a patient having a myocardial infarction. The health care provider prescribes an IV infusion of alteplase. Which intervention would the nurse include during the administration of this medication? Monitor for changes in neurologic status. Assess BP for orthostatic changes. Apply a pressure dressing to IV insertion sites. Start additional IV lines after alteplase infusion has begun

Monitor for changes in neurologic status. Assessment for changes in neurologic status is the priority nursing intervention because this may indicate a cerebral bleed during the thrombolytic infusion. Monitoring BP for orthostatic changes is necessary with the use of short-acting nitrates. Application of a pressure dressing to the IV insertion site is not done unless evidence of bleeding is noted. Additional IV therapy lines should be inserted before the alteplase begins. Perform all other invasive procedures before giving the thrombolytic agent to reduce the risk for bleeding.

The nurse is preparing an initial care plan for a patient who presents with chest pain. Which is the priority nursing intervention? Monitoring the patient's cardiac rhythm Discussing the losses associated with chronic illness Encouraging verbalization of feelings, perceptions, and fears Advising the patient to avoid heavy meals and extreme weather conditions

Monitoring the patient's cardiac rhythm A patient with chest pain may have acute coronary syndrome. The priority is to stabilize the patient, determine the plan of care, and prevent complications. Ongoing care should include continuous electrocardiogram (ECG) monitoring. The nurse should help the patient with anxiety and stress to work on the losses due to chronic illness to prevent sudden depression-related cardiac workload, but that is not the highest priority. A patient with chronic stable angina is advised to avoid heavy meals and extreme weather to reduce the probability of symptoms, and this may be appropriate to teach at a later time. The nurse should encourage verbalization of feelings, perceptions, and fears that increases workload on heart but that is not a higher priority than monitoring for life-threatening dysrhythmias.

The nurse assesses a patient with diabetes who reports shortness of breath, neck pain, and hypoglycemic symptoms. The patient's BP is 130/86 mm Hg, heart rate is 102 beats/min, respiratory rate is 24 breaths/min, and the finger-stick blood glucose is 136 mg/dL. Which condition would the nurse suspect? Myocardial infarction (MI) Late-stage diabetic ketoacidosis Early-onset diabetic ketoacidosis Hyperosmolar hyperglycemic nonketotic syndrome

Myocardial infarction (MI) Signs and symptoms of an MI include shortness of breath, neck pain, and cool, clammy skin. Although cool, clammy skin may resemble a hypoglycemic reaction, when found along with shortness of breath and neck pain it is very specific for an MI. The patient is not experiencing a complication of diabetes (ketoacidosis or hyperosmolar hyperglycemic nonketotic syndrome). The blood glucose is close to normal, and further diagnostics would be required to determine a diabetic complication.

A patient with chronic stable angina asks the nurse about the origin of the pain. Which condition would the nurse explain as the cause of the pain? Atherosclerosis Myocardial ischemia Movement of a thromboembolus Coronary artery vasoconstriction

Myocardial ischemia The pain of angina is caused by an inadequate oxygen supply to the myocardium, resulting in ischemia. Atherosclerosis can eventually lead to myocardial ischemia. A thromboembolus can cause pain, but it is not stable angina. Coronary artery vasoconstriction is the cause of Prinzmetal's angina.

A patient reports episodic chest pain lasting a few minutes that is provoked by exertion and relieved within 10 minutes upon resting. Which cause of the patient's symptoms would the nurse suspect? Spasm of a major coronary artery Exposure of the thrombogenic surface to plaque Myocardial ischemia due to coronary artery disease Myocardial ischemia secondary to microvascular disease

Myocardial ischemia due to coronary artery disease Pain is provoked upon exertion and relieved upon rest due to an increase in demand for oxygen or a decrease in the supply of oxygen that leads to myocardial ischemia secondary to coronary artery disease. Coronary vasospasm leads to Prinzmetal's angina, which primarily occurs at rest. Unstable angina occurs due to the rupture of thickened plaque, exposing thrombogenic surfaces in blood vessels. Microvascular angina is common in women and results from myocardial ischemia secondary to microvascular disease.

A patient with a diagnosis of unstable angina is admitted to the intensive care unit. Which drug therapies will the nurse anticipate to be prescribed? Select all that apply. Nitrates Antiplatelet therapy Anticoagulant therapy β-adrenergic blockers Angiotensin-converting enzyme (ACE) inhibitors

Nitrates Antiplatelet therapy Anticoagulant therapy Nitrates are the first line of drug therapy for angina because of their mechanisms of dilating peripheral blood vessels to reduce cardiac workload and dilating the coronary arteries and collateral vessels to increase blood flow to ischemic areas of the heart. Antiplatelet therapy works in different ways to inhibit platelet activation and aggregation. Anticoagulants have several different mechanisms of action to prevent the formation of fibrin and thrombin and interfere with formation of clotting factors. β-adrenergic blockers are commonly used in the treatment of chronic stable angina and acute coronary syndrome. ACE inhibitors are used for heart failure, tachycardia, myocardial infarction, hypertension, diabetes, and chronic kidney disease.

A patient with a history of unstable angina reports a sudden onset of retrosternal chest heaviness and tightness, fatigue, shortness of breath, and nausea. Which actions would the nurse take? Select all that apply. Obtain a 12-lead electrocardiogram (ECG). Administer sublingual nitroglycerin. Place the patient in a supine position. Apply high-flow oxygen by face mask. Auscultate for a pericardial friction rub.

Obtain a 12-lead electrocardiogram (ECG). Administer sublingual nitroglycerin. Initial management of the patient with chest pain includes the following: Obtain a 12-lead ECG and start continuous ECG monitoring. Position the patient in an upright, not supine, position unless contraindicated, and initiate oxygen by nasal cannula (not high-flow by face mask) to keep oxygen saturation above 93%. Establish an IV route to provide an access for emergency drug therapy. Give sublingual nitroglycerin and aspirin (chewable). Morphine sulfate is given for pain unrelieved by nitroglycerin (NTG). Auscultating for a pericardial friction rub is not an appropriate action.

A 74-yr-old man with a history of prostate cancer and hypertension is admitted to the emergency department with substernal chest pain. Which priority action will the nurse complete before administering sublingual nitroglycerin? Administer morphine sulfate IV. Auscultate heart and lung sounds. Obtain a 12-lead electrocardiogram (ECG). Assess for coronary artery disease risk factors.

Obtain a 12-lead electrocardiogram (ECG). Rationale: If a patient has chest pain, the nurse should institute the following measures: (1) administer supplemental oxygen and position the patient in upright position unless contraindicated, (2) assess vital signs, (3) obtain a 12-lead ECG, (4) provide prompt pain relief first with a nitrate followed by an opioid analgesic if needed, and (5) auscultate heart sounds. Obtaining a 12-lead ECG during chest pain aids in the diagnosis.

The nurse is examining the electrocardiogram (ECG) of a patient just admitted with a suspected myocardial infarction (MI). Which ECG change is most indicative of prolonged or complete coronary occlusion? 1. Sinus tachycardia 2. Pathologic Q wave 3. Fibrillatory P waves 4. Prolonged PR interval

Pathologic Q wave Rationale: The presence of a pathologic Q wave, as often accompanies STEMI, is indicative of complete coronary occlusion. Sinus tachycardia, fibrillatory P waves (e.g., atrial fibrillation), or a prolonged PR interval (first-degree heart block) are not direct indicators of extensive occlusion.

The nurse is caring for four male patients in a health care facility. Which patient has the highest risk for developing coronary artery disease (CAD)? Patient 1 Patient 2 Patient 3 Patient 4

Patient 3 Obesity, high serum cholesterol, low high-density cholesterol, and diabetes mellitus increase the risk of CAD. Obesity is measured using body mass index (BMI), which is calculated as weight/height2. The acceptable BMI range is 18.5 to 24.9 kg/m2, serum cholesterol is 40 mg/dL, and glycosylated hemoglobin (Hb A1C) is 2%; Patient 3's serum cholesterol is 204 mg/dL, HDL cholesterol is 38 mg/dL, and Hb A1C is 7.4%. Therefore Patient 3 has the highest risk for acquiring coronary artery disease. Patients 1, 2, and 4 have normal BMIs, serum cholesterol levels, HDL levels, and Hb A1C levels. These patients are not are risk for developing CAD.

The nurse is caring for patients who are receiving treatment to reduce lipoproteins and cholesterol. Which findings describe a patient who would experience intensified effects from prescribed warfarin? Patient A Patient B Patient C Patient D

Patient B Patient B may be taking a fibric acid derivative treatment, such as fenofibrate, because its side effects are rashes, mild gastrointestinal disturbances, and elevated liver enzymes. Fibric acid derivatives may increase the effects of warfarin for Patient B. Patient A has pain in the knee joint with no inflammation, which is called arthralgia, a side effect of omega-3 fatty acids. Patient C has rash on the skin, gastrointestinal disturbances, and elevated liver enzymes along with myopathy and rhabdomyolysis. These are side effects of HMG-CoA reductase inhibitors, such as atorvastatin. Patient D has pruritus and flushing in the upper torso with gastrointestinal disturbances, orthostatic hypotension, and elevated homocysteine levels, which are side effects of niacin.

The nurse reviews the medical records of four patients with chest pain. Which patient would likely benefit from nitroprusside and intraaortic balloon pump (IABP) therapy? Patient A Patient B Patient C Patient D

Patient C Showing symptoms of dyspnea, pulmonary edema, and decreased cardiac output, Patient C may have papillary muscle dysfunction and is at risk for clinical deterioration. This patient must be given nitroprusside and IABP therapy to reduce the afterload on the heart. Patient A may have Dressler syndrome, which is treated by short-term corticosteroids. Patient B is exhibiting signs of acute pericarditis, which is treated by aspirin and nonsteroidal antiinflammatory agents. Patient D may have heart failure and may require intensive care.

The nurse reviews the medication profiles of four patients. Which patient has been prescribed a medication that the nurse would question? Patient A Patient B Patient C Patient D

Patient D Ranolazine is a sodium-current inhibitor used to treat chronic angina. It is known to prolong the QT interval, as is fluoxetine, so the medication regimen of Patient D needs correction. Patient A is intolerant of angiotensin-converting enzyme (ACE) inhibitors; therefore the patient can take an angiotensin II receptor blocker such as losartan. Patient B has a contraindication to the β-blocker; therefore the patient can take nadolol or can use a calcium channel blocker such as felodipine. For Patient C, a calcium channel blocker, such as felodipine, and a nitrate, such as isosorbide dinitrate, are used in the treatment of Prinzmetal's angina.

Which patient will the nurse expect to receive a prescription for cholestyramine? Patient with a triglyceride level of 138 mg/dL Patient with a total cholesterol level of 180 mg/dL Patient with high-density lipoprotein (HDL) cholesterol of 60 mg/dL Patient with low-density lipoprotein (LDL) cholesterol of 190 mg/dL

Patient with low-density lipoprotein (LDL) cholesterol of 190 mg/dL Patients with LDL cholesterol levels greater than 160 mg/dL are at a risk for acquiring coronary artery disease and would benefit from receiving cholestyramine. Patients with a triglyceride level of 138 mg/dL are not at a risk for coronary heart disease, and cholestyramine is not indicated. Patients with total cholesterol levels less than 200 mg/dL do not require drug treatment because they are not at risk of developing coronary heart disease. Patients with HDL less than 40 mg/dL are at a risk of developing coronary heart disease and must be treated with appropriate drugs, but cholestyramine does not work on HDL.

Diagnostic tests confirm that a patient is experiencing a ST-segment-elevation myocardial infarction (STEMI). For which first line of treatment will the nurse prepare the patient? Antiplatelet and statins therapy Dual antiplatelet therapy and heparin Transmyocardial laser revascularization Percutaneous coronary intervention (PCI)

Percutaneous coronary intervention (PCI) Emergent PCI is the first line of treatment for patients with confirmed STEMI (i.e., ST-elevation on the electrocardiogram [ECG] and/or positive cardiac biomarkers). The goal is to open the blocked artery within 90 minutes of arrival to a facility that has an interventional cardiac catheterization laboratory. Antiplatelet and statins therapy improves vein graft patency in a patient who has undergone coronary artery bypass grafting (CABG) involving the saphenous vein. Dual antiplatelet therapy and heparin will help a patient with ongoing angina and negative cardiac markers. Transmyocardial laser revascularization is used for a patient with advanced coronary artery disease and persistent angina even after maximum medical therapy.

The nurse is caring for a patient two days after the patient experienced a myocardial infarction. For which complications will the nurse monitor the patient? Select all that apply. Pericarditis Heart failure Dysrhythmia Dressler syndrome Cardiogenic shock

Pericarditis Heart failure Dysrhythmia Cardiogenic shock Dysrhythmia, heart failure, pericarditis, and cardiogenic shock are all serious complications that can arise immediately and several days after myocardial infarction. Dressler syndrome is also a possible complication post myocardial infarction but presents as pericarditis with effusion and fever that develops four to six weeks later.

A patient with ventricular dysfunction receives a prescription for an angiotensin-converting enzyme (ACE) inhibitor. The nurse should monitor the patient for which indication of medication intolerance? Constipation Hyperactivity Persistent cough Decreased potassium levels

Persistent cough A nonproductive and persistent cough may occur in 5% to 25% of individuals. It may take up to two weeks or longer for coughing to subside after the ACE inhibitor is discontinued. If one ACE inhibitor causes coughing, then it is likely that the others will also. Other side effects of ACE inhibitors include diarrhea (versus constipation), drowsiness (versus hyperactivity), and elevated (versus decreased) blood potassium levels.

The nurse provides basic exercise guidelines for a patient who is being discharged following acute coronary syndrome (ACS). Which guidelines following the FITT formula would the nurse recommend? Physical activity that is regular, rhythmic, and repetitive An increase in heart rate of 25 beats/min over the resting heart rate (HR) Physical activity sessions that begin at 5 to 10 minutes and build up to 45 minutes Mild weightlifting for 10 minutes to allow stretching of muscles before exercising

Physical activity that is regular, rhythmic, and repetitive The FITT formula includes physical activities that are regular, rhythmic, and repetitive, using large muscles to build up endurance. The increase in HR should not exceed 20 beats/min over the resting HR. The physical activity session should build up to 30 minutes. There is no weightlifting included in the formula.

Which characteristic describes a complicated coronary artery lesion? Formation of a fibrous plaque Lipid-filled smooth muscle cells Platelet aggregation and adhesion Transport of lipids into arterial intima

Platelet aggregation and adhesion A complicated lesion is characterized by accumulation of platelets leading to thrombus formation. Formation of a fibrous plaque is the beginning of progressive changes in the endothelium of the arterial wall. It is called the fibrous plaque stage. Fatty streaks are the earliest lesions of atherosclerosis and are characterized by lipid-filled smooth muscle cells. Transport of lipids into the arterial intima occurs in the fibrous plaque stage.

A patient experiences anginal pain. Which intervention would the nurse use? Encourage the patient to perform isometric exercises. Place the patient in recumbent position during the attack. Position the patient upright and give supplemental oxygen. Recommend a salt-rich diet to prevent orthostatic hypotension.

Position the patient upright and give supplemental oxygen. A patient with angina should be placed in an upright position and supplied oxygen to provide comfort and to attain an appropriate amount of oxygen in blood unless contraindicated. Isometric exercises are stressful and may exacerbate the symptoms by increasing the cardiac workload. A recumbent positioning of the patient may increase the discomfort. Salt and saturated fat foods are restricted in the patient to prevent further complications.

A patient has received a bolus dose and an infusion of alteplase (Activase) for an ST-segment elevation myocardial infarction (STEMI). Which patient assessment would determine the effectiveness of the medication? 1. Presence of chest pain 2. Blood in the urine or stool 3. Tachycardia with hypotension 4. Decreased level of consciousness

Presence of chest pain Rationale: Alteplase is a fibrinolytic agent that is administered to patients who have had a STEMI. If the medication is effective, the patient's chest pain will resolve because the medication dissolves the thrombus in the coronary artery and results in reperfusion of the myocardium. Bleeding is a major complication of fibrinolytic therapy. Signs of major bleeding include decreased level of consciousness, blood in the urine or stool, and increased heart rate with decreased blood pressure.

Which type of angina occurs as a result of coronary vasospasm? Nocturnal angina Prinzmetal's angina Microvascular angina Chronic stable angina

Prinzmetal's angina Prinzmetal's angina is chest pain that occurs as a result of coronary vasospasm. Nocturnal angina occurs at night, during sleep. Microvascular angina is a result of myocardial ischemia secondary to microvascular disease, affecting the small, distal branches of coronary arteries. Chronic stable angina is the result of myocardial ischemia caused by an oxygen supply/demand mismatch.

A patient with chronic stable angina shows a prolonged QT interval on the electrocardiogram (ECG) monitor. Which medication on the list of drugs prescribed for the patient would the nurse question? Aspirin Heparin Captopril Ranolazine

Ranolazine Ranolazine is a sodium-current inhibitor that further prolongs the QT interval in patients who have QT prolongation. Aspirin and heparin are safe in patients with chronic stable angina and are used to inhibit clot formation in blood vessels. Captopril is an angiotensin II-converting inhibitor that reduces the risk of cardiac events by dilating the blood vessels and reducing BP.

A patient is prescribed a statin drug to decrease levels of low-density lipoproteins and triglycerides. For which symptoms would the nurse teach the patient to observe? Select all that apply. Rash Pruritus Flushing Muscle pain Muscle weakness Gastrointestinal disturbances

Rash Muscle pain Muscle weakness Gastrointestinal disturbances Statin drugs have been found to lower low-density lipoproteins and triglycerides. Common side effects of this class of drugs include rash, myopathy, rhabdomyolysis, and gastrointestinal disturbance, as well as elevated enzyme levels. Flushing and pruritus in the upper torso and face have not been cited as a side effect of statins but may be seen with the use of niacin.

daily intake of calories and diet

Recommended: 5%-35% of total daily calories (including <7% from saturated fat) Combined total fat calories from: • Saturated fats: Limit fats that are usually solid at room and refrigerator temperature --lard -butter -whole-milk products -fatty cuts of meat -bacon • Trans fats: Mainly in foods made with hydrogenated vegetable oils, such as many hard margarines and shortenings • Unsaturated fats: In oils that are usually liquid at room and refrigerator temperature -olive -corn -sunflower -soybean 2 types of unsaturated fats: Monounsaturated fats: In greatest amounts in foods from plants: -olives -avocadoes -canola, sunflower, and peanut oils. Polyunsaturated fats: Found mostly in -nuts -seeds -fish -seed oils -oysters Omega-3 fatty acid is a type of polyunsaturated fat that may help reduce the risk of CAD. Cholesterol <200 mg Plant stanols or sterols (e.g., margarines, nuts, seeds, legumes, vegetable oils) -2 g Dietary soluble fiber -10-25 g of soluble fiber Total calories Only enough calories to reach or maintain a healthy weight Physical Activity -30 min of a moderate intensity physical activity, such as brisk walking, on most, and preferably all, days of the week.

The nurse provides health promotion teaching to a patient recently diagnosed with coronary artery disease (CAD). Which information would the nurse include? Select all that apply. Reduce total fat intake. Decrease the amount of fiber in the diet. Increase complex carbohydrates in the diet. Perform isometric exercises at least 10 minutes daily. Perform moderate physical activity at least 30 minutes five days a week

Reduce total fat intake. Increase complex carbohydrates in the diet. Perform moderate physical activity at least 30 minutes five days a week. Dietary modifications should include a decrease in total fat intake, as well as an increase in complex carbohydrates (whole grains, fruits, and vegetables). A moderate physical activity program should include isotonic exercises, such as walking, hiking, or jogging performed for at least 30 minutes on most days (e.g., at least five days a week). Modifications should include an increase in fiber. Isometric exercises, such as weightlifting, are recommended to increase muscle strength two days per week only.

A patient is scheduled for coronary artery bypass graft (CABG) surgery. Which type of graft, if used, would require postoperative antiplatelet and statin therapy to improve graft patency? Radial artery graft Saphenous vein graft Gastroepiploic artery graft Internal mammary artery graft

Saphenous vein graft CABG involves the replacement of conduits that transport blood between the aorta and the coronary artery. A patient with a saphenous vein graft may develop intimal hyperplasia, which contributes to stenosis and graft occlusions. This patient should receive antiplatelet therapy and statins after surgery to improve vein graft patency. A patient with radial artery CABG should receive calcium channel blockers and long-acting nitrates to control coronary spasms. A patient with gastroepiploic artery CABG generally has a high graft patency rate; the chances of graft-related problems are very rare. A patient with CABG of the internal mammary artery has an average graft patency of more than 90%, even after 10 years. A patient with high graft patency rates may not require any medication to maintain the patency of the graft.

A patient has a family history of myocardial infarction (MI). Which data indicates to the nurse that the patient has additional risk factors for coronary artery disease (CAD)? Select all that apply. Smoking cessation three years ago Serum cholesterol level of 260 mg/dL Fasting triglyceride level of 110 mg/dL High level of anxiety over last two years related to family issues Works over 50 hours/week in an office with little time to exercise Lives with an adult child who smokes two packs of cigarettes per day

Serum cholesterol level of 260 mg/dL High level of anxiety over last two years related to family issues Works over 50 hours/week in an office with little time to exercise Lives with an adult child who smokes two packs of cigarettes per day Serum cholesterol level greater than 200 mg/dL is a risk factor for CAD. Stress is an additional risk factor for developing CAD. Secondhand smoke increases the risk of CAD. A sedentary job and lack of exercising are risk factors for CAD. Smoking cessation will lead to a reduced mortality rate after a period of 12 months. Fasting triglyceride levels above 150 mg/dL are a risk factor for CAD.

Which behaviors can help reduce psychologic risk factors that contribute to the development of coronary artery disease (CAD)? Select all that apply. Monitoring glucose levels daily Setting realistic goals for exercise Changing eating patterns and habits Planning time for adequate rest and sleep Increasing activities to a prescribed fitness level Learning effective stress management techniques

Setting realistic goals for exercise Planning time for adequate rest and sleep Learning effective stress management techniques Setting realistic goals for exercise increases psychologic well-being because reaching attainable goals can help boost the patient's level of confidence. Planning time for adequate rest and sleep contributes to the patient's psychologic well-being, which can reduce the risk for development of CAD. Learning effective stress management techniques helps to reduce the risk for development of CAD by helping the patient better manage stress levels. Increasing activities to a prescribed fitness level, monitoring glucose levels daily, and managing eating patterns and habits are all appropriate behaviors to aid in the reduction of risk factors for CAD, but they address physiologic, not psychologic, components.

Which lipid-lowering medication has adverse effects including muscle pain, elevated liver enzymes, and elevated creatine kinase levels? Niacin Colestipol Simvastatin Gemfibrozil

Simvastatin Muscle pain and elevated creatine kinase levels are manifestations of rhabdomyolysis. Elevated liver enzymes and rhabdomyolysis are adverse effects of statin drugs, such as simvastatin. Side effects of niacin include pruritus and flushing; high doses may cause decreased liver function. Colestipol is a bile-acid sequestrant drug. This drug does not have any major adverse effects except that it lowers the absorption of drugs such as warfarin, digoxin, and thiazide diuretics. Gemfibrozil is a fibric acid derivative that can cause rhabdomyolysis when given with a statin drug.

A patient is scheduled for a minimally invasive direct coronary artery bypass (MIDCAB). Which steps are involved in the procedure? Select all that apply. A robot is used to replace the mitral valve. Small incisions are made between the ribs. Cardiac catheterization is performed during the procedure. A mechanical stabilizer is placed to immobilize the operative site. A thoracoscope or robotic assistance is used to free the internal mammary artery.

Small incisions are made between the ribs. A mechanical stabilizer is placed to immobilize the operative site. A thoracoscope or robotic assistance is used to free the internal mammary artery. MIDCAB offers patients with disease of the left anterior descending or right coronary artery an approach to surgical treatment that does not involve a sternotomy and cardiopulmonary bypass (CPB). It involves several small incisions between the ribs to dissect the internal mammary artery (IMA) with a thoracoscope or with robotic assistance. The heart is then slowed or stopped temporarily with adenosine, which is assisted by a mechanical stabilizer to immobilize the operative site. The IMA is then sutured to the coronary artery. A robot is used to replace the mitral valve during robot-assisted cardiothoracic surgery. Transmyocardial laser revascularization involves cardiac catheterization. Mechanical stabilizers are used on a beating heart during off-pump coronary artery bypass.

Which information would the nurse include when teaching a patient about risk factors related to Prinzmetal's angina? Diabetes Smoking Postmenopausal state High cholesterol level

Smoking Prinzmetal's angina occurs at rest due to coronary artery spasm. It is usually due to spasm of a major coronary artery. Factors contributing to coronary artery spasm include increased myocardial oxygen demand and increased levels of certain substances such as tobacco smoke, alcohol, and cocaine. Patients with diabetes have an increased prevalence of silent ischemia. Microvascular angina is more common in postmenopausal women. High cholesterol is not a risk factor for Prinzmetal's angina.

Which complementary lipid-lowering agents may be recommended to a patient for reduction of total cholesterol? Select all that apply. Soy Psyllium Flaxseed Plant sterols Red yeast rice Omega-3 fatty acids

Soy Psyllium Plant sterols Red yeast rice There is scientific evidence that plant sterols, soy, red yeast rice, and psyllium all contribute to the total reduction of cholesterol. There is conflicting evidence on the efficacy of flaxseed for reducing total cholesterol. Omega-3 fatty acids are responsible for the reduction of triglyceride levels, but not total cholesterol.

A patient is admitted to the intensive care unit in stable condition with a diagnosis of myocardial infarction. Which common medications will be used to treat this condition? Select all that apply. Diuretics Stool softeners Prophylactic antibiotics Dual antiplatelet therapy IV nitroglycerin Low -molecular-weight heparin (LMWH)

Stool softeners Dual antiplatelet therapy IV nitroglycerin Low -molecular-weight heparin (LMWH) After an MI, the patient may be predisposed to constipation because of bed rest and opioid drugs. Stool softeners (e.g., docusate sodium [Colace]) prevent straining and the resultant vagal stimulation from the Valsalva maneuver. Vagal stimulation produces bradycardia and can provoke dysrhythmias. Drug therapy for myocardial infarction includes IV nitroglycerin, dual antiplatelet therapy (e.g., aspirin and clopidogrel), and systemic anticoagulation with either LMWH given subcutaneously or IV unfractionated heparin, which are the initial drug treatments of choice for acute coronary syndrome (ACS). Diuretics and prophylactic antibiotics are not appropriate at this time.

In which order would a patient experience progression of coronary artery disease (CAD)? Thrombus formation occurs. Fibrous plaque narrows the vessel lumen. Streaks of fat develop within the smooth muscle cells. Collateral circulation is formed; new blood circulation routes are created or utilized.

Streaks of fat develop within the smooth muscle cells. Fibrous plaque narrows the vessel lumen. Thrombus formation occurs. Collateral circulation is formed; new blood circulation routes are created or utilized. The development stages of CAD are the fatty streaks stage, fibrous plaque stage, and, finally, the complicated lesion stage with thrombus formation. If the blockage of coronary arteries occurs slowly over time, then there is a chance of collateral circulation occurring, in which new routes of blood circulation are formed to bypass blockages.

A 55-year-old patient with cardiovascular disease (CVD) is prescribed 81 mg of aspirin daily. Which information in the patient's medical history would be a contraindication to aspirin requiring the nurse to discuss the prescription with the provider? Select all that apply. Age Postmenopause Stroke two years ago History of gastrointestinal bleeding Calculated 10-year CVD risk of 10%

Stroke two years ago History of gastrointestinal bleeding Aspirin is an antiplatelet drug that can be used to prevent the development of coronary artery disease. It prevents the aggregation of platelets, which can prevent plaque from increasing in size. Due to the increased risk of bleeding, a history of stroke and/or a history of gastrointestinal bleeding are contraindications. Low-dose aspirin is recommended for adults age 50 to 59 years old who have a calculated 10-year CVD risk of 10% or more and are not at increased risk of bleeding. Postmenopause is not a contraindication.

A patient returns to the unit after a cardiac catheterization. Which nursing care would the registered nurse delegate to the unlicensed assistant personnel (UAP)? Take vital signs and report any abnormal values. Check for bleeding at the catheter insertion site. Prepare discharge teaching related to complications. Monitor the electrocardiogram for S-T segment changes.

Take vital signs and report any abnormal values. Rationale: Vital signs should be delegated to the UAP. Assessment of the site, preparation of discharge teaching, and monitoring for S-T elevation would be registered nurse scope of practice.

A patient asks the nurse about resuming sexual activity after acute coronary syndrome (ACS). Which information would the nurse include in the patient's teaching plan? Take a hot shower just before intercourse to provide relaxation. Limit the time, including foreplay, to 30 minutes to prevent overexertion. Wait an hour after ingesting a large meal before engaging in sexual activity. Taking a prophylactic nitrate may decrease chest pain during sexual activity.

Taking a prophylactic nitrate may decrease chest pain during sexual activity. Taking a prophylactic nitrate may decrease chest pain during sexual activity. Hot or cold showers should be avoided just before or after intercourse. Consumption of food and alcohol should be reduced before intercourse is anticipated (e.g., waiting three to four hours after ingesting a large meal before engaging in sexual activity). There is no established time limit. Foreplay is desirable because it allows a gradual increase in heart rate before orgasm.

A male patient with coronary artery disease (CAD) has a low-density lipoprotein (LDL) cholesterol of 98 mg/dL and high-density lipoprotein (HDL) cholesterol of 47 mg/dL. What information should the nurse include in patient teaching? 1. Consume a diet low in fats. 2. Reduce total caloric intake. 3. Increase intake of olive oil. 4. The lipid levels are normal.

The lipid levels are normal. Rationale: For men, the recommended LDL is less than 100 mg/dL, and the recommended level for HDL is greater than 40mg/dL. His normal lipid levels should be included in the patient teaching and encourage him to continue taking care of himself. Assessing his need for teaching related to diet should also be done.

A patient with hyperlipidemia has been prescribed niacin. For which reason would the nurse administer aspirin 30 minutes before administering niacin? To relieve pain To prevent skin flushing To prevent blood clotting To prevent atherosclerotic plaque

To prevent skin flushing Flushing of the face and neck is a common side effect of niacin. Premedicating the patient with aspirin reduces flushing. Aspirin is an analgesic and antiplatelet medication. Preventing platelet aggregation helps to prevent the formation of atherosclerotic plaque. Although aspirin has all of these effects, in this case the most likely reason for the nurse to administer this drug is to relieve flushing.

For which reason would an everolimus coating be used in a coronary stent? To avoid patient noncompliance to oral drugs To promote patency of the affected blood vessel To reduce the risk of overgrowth for the intimal lining within the stent To decrease the dose of dual antiplatelet drugs needed to prevent stent thrombosis

To reduce the risk of overgrowth for the intimal lining within the stent Stents may be coated with certain medications such as everolimus. These medications help to check the overgrowth of new intima over the stent wall, thus preventing restenosis. Drug-eluting stents do not help in promoting drug compliance or promoting the patency of the affected blood vessel. The dose of dual antiplatelet therapy is not dependent on the type of stent used.

A nurse plans to provide teaching to a patient about smoking cessation to decrease the risk for coronary artery disease (CAD). Which rationale supports this intervention? Select all that apply. Tobacco smoking increases estrogen levels. Tobacco smoking decreases BP. Tobacco smoke increases the low-density lipoprotein levels. Nicotine in tobacco smoke causes release of catecholamines. Carbon monoxide in tobacco smoke affects the oxygen-carrying capacity of hemoglobin.

Tobacco smoke increases the low-density lipoprotein levels. Nicotine in tobacco smoke causes release of catecholamines. Carbon monoxide in tobacco smoke affects the oxygen-carrying capacity of hemoglobin. Nicotine present in tobacco smoke stimulates the release of catecholamines. Catecholamines have a stimulatory effect on the sympathetic nervous system that causes an increase in heart rate and BP. Carbon monoxide present in tobacco smoke has a greater affinity to hemoglobin than does oxygen. Therefore carbon monoxide reduces the oxygen-carrying capacity of blood. Tobacco smoke is known to increase the level of low-density lipoproteins and subsequently a decrease in high-density lipoproteins. All of these factors can lead to atherosclerosis. Tobacco smoke is known to decrease estrogen levels in premenopausal women, thereby increasing their susceptibility for coronary artery disease.

Which findings in a patient's medical history are major modifiable risk factors for coronary artery disease (CAD)? Select all that apply. Age Gender Tobacco use BP of 150/92 mm Hg Genetic predisposition Waist circumference 42 inche

Tobacco use BP of 150/92 mm Hg Waist circumference 42 inches Major modifiable risk factors include BP >140/90 mm Hg, tobacco use, and waist circumference >40 inches in men and >35 inches in women. Age, genetic predisposition, and gender are nonmodifiable.

Which foods would the nurse encourage patients at risk for coronary artery disease (CAD) to include in their diets? (Select all that apply.) 1. Tofu 2. Walnuts 3. Tuna fish 4. Whole milk 5. Orange juice

Tofu Walnuts Tuna fish Rationale: Tuna fish, tofu, and walnuts are all rich in omega-3 fatty acids, which have been shown to reduce the risks associated with CAD when consumed regularly.

Lipid-Lowering Drug Therapy

Treatment of high cholesterol focuses mostly on lowering LDL cholesterol. A complete lipid profile -should be done every 5 years -beginning at age 20 -Middle-aged adults should be screened every 1 to 2 years Guidelines the following people receive statin therapy: (1) patients with known CVD (2) patients with primary elevations of LDL cholesterol levels of 190 mg/dL or greater (e.g., familial hypercholesterolemia) (3) patients 40 to 75 years old with diabetes and LDL cholesterol levels between 70 and 189 mg/dL (4) patients 40 to 75 years old with LDL cholesterol levels between 70 and 189 mg/dL and a 10-year risk for CVD of at least 7.5%.8 Drug therapy should continue for a lifetime unless the patient becomes intolerant. In that case, the patient should switch to another drug Concurrent diet change is essential Treatment also includes -weight loss -increased physical activity Serum lipid levels should be reassessed after 6 weeks of therapy.

A patient's history includes an ejection fraction of 30%, diabetes, reports of stabbing chest pain that is relieved upon rest, and prescription for captopril. If the patient is intolerant of captopril, which medication would the nurse expect will be prescribed? Valsartan Tirofiban Everolimus Carvedilol

Valsartan A patient with an ejection fraction of 30%, diabetes mellitus, and angina has a cardiovascular risk. The patient who is intolerant of an angiotensin-converting enzyme (ACE) inhibitor should receive an angiotensin II receptor blocker such as valsartan. Tirofiban is a glycoprotein IIb/IIIa inhibitor that is used to prevent the abrupt closure of the stents during percutaneous coronary intervention (PCI). Everolimus prevents the overgrowth of new intima that leads to stent restenosis. Carvedilol is a β-blocker that carries cautions for its use in a patient with diabetes because it masks the signs of hypoglycemia.

A patient reports cyclic and short bursts of pain in the chest during the day and night that are relieved by mild exercise. Which medication would the nurse anticipate will be prescribed? Prasugrel Sirolimus Abciximab Verapamil

Verapamil Cyclic and short bursts of pain in the chest during the day and night indicate Prinzmetal's angina. Calcium channel blockers, such as verapamil and nitrates, are useful to control this angina. Prasugrel is an antiplatelet agent used in the treatment of unstable angina. Sirolimus is used in drug-eluting stents to prevent stent restenosis by preventing the overgrowth of new intima. Abciximab is a glycoprotein IIb/IIIa inhibitor that helps prevent the binding of fibrinogen to platelets, thereby blocking platelet aggregation.

Which medications may relieve spasms associated with radial artery coronary artery bypass surgery? Select all that apply. Captopril Carvedilol Verapamil Nicardipine Tenecteplase

Verapamil Nicardipine The radial artery is a thick muscular artery that is prone to spasm. A patient with coronary revascularization using the radial artery is administered long-acting nitrates and calcium channel blockers. Nicardipine and verapamil are calcium channel blockers that will help to control the spasms. Captopril, an angiotensin-converting enzyme (ACE) inhibitor, prevents ventricular remodeling and prevents the progression of heart failure in a patient with elevated ST-segment myocardial infarction. Carvedilol, a β-blocker, helps reduce workload on the heart. Tenecteplase stops the infarction in a patient with myocardial infarction (MI) by dissolving the thrombus and reperfusing the myocardium.

The nurse is caring for a patient with chronic stable angina who has a history of a stroke. Which prescription would the nurse question? Aspirin Vorapaxar Metoprolol Nitroglycerin

Vorapaxar The nurse should be concerned about the prescription for vorapaxar. It must not be used in people who have had a stroke because the risk for bleeding in the head is too great. Aspirin, nitroglycerin, and metoprolol may be appropriate for treatment of chronic stable angina in a patient who has had a stroke.

Which sources of polyunsaturated fat would the nurse include in a teaching plan for a patient with coronary artery disease (CAD)? Select all that apply. Walnuts Palm oil Egg yolk Soybeans Sour cream

Walnuts Soybeans Polyunsaturated fats are found in greatest amounts in many kinds of nuts and in foods from plants including safflower, sunflower, corn, soybean, and cottonseed oils. Palm oil, egg yolk, and sour cream are major dietary sources of saturated fat.

Which question would the nurse ask the older patient regarding risk factors for coronary artery disease (CAD)? What is your body mass index (BMI) number? Did you receive a pneumococcal vaccine? When did you last have a bowel movement? Did you travel abroad within the last 12 months?

What is your body mass index (BMI) number? Risk for CAD increases with obesity, which is defined as a BMI of more than 30 kg/m2. Constipation is not a risk factor for CAD. Traveling abroad to underdeveloped countries is a risk factor for infectious disease but not for CAD. Pneumococcal vaccine protects the elderly patient from pneumonia but not from CAD.

acute coronary syndrome (ACS)

When chest pain from ischemia is prolonged and not immediately reversible. Includes spectrum of -non-ST elevation acute coronary syndrome (Unstable angina and non-ST-segment-elevation myocardial infarction [NSTEMI]) -ST-segment-elevation myocardial infarction (STEMI) ACS is caused by the decline of a once-stable atherosclerotic plaque. The previously stable plaque ruptures, releasing the lipid core into the vessel. This causes platelet aggregation and thrombus formation. The vessel may be partially blocked by a thrombus (manifesting as UA or NSTEMI) or totally blocked by a thrombus (manifesting as STEMI). We are not sure what causes the plaque to suddenly become unstable. Systemic inflammation may play a role.

The community health nurse is planning health promotion teaching to prevent coronary artery disease (CAD). Which population has the highest incidence of CAD? White male Hispanic male Black male Native American female

White male. The incidence of CAD and myocardial infarction (MI) is highest among white, middle-aged men. Hispanic individuals have lower rates of CAD than non-Hispanic whites or blacks. Blacks have an earlier age of onset and more severe CAD than whites and more than twice the mortality rate of whites of the same age. Native Americans have increased mortality in 35-year-old-and-under people and have major modifiable risk factors, such as diabetes.

A patient is admitted to the ICU with a diagnosis of NSTEMI. Which drugs(s) would the nurse expect the patient to receive? (select all that apply) a. Oral statin therapy b. Antiplatelet therapy c Thrombolytic therapy d. Prophylactic antibiotics e. Intravenous nitroglycerin

a, b, e Rationale: When a patient presents with suspected ACS, antiplatelet therapy, IV NTG, and atorvastatin are drug treatments of choice. For patients with UA and NSTEMI, heparin (UH or LMWH) is recommended to prevent microemboli from forming and causing further chest pain. DAPT (e.g., aspirin and clopidogrel or ticagrelor [Brilinta]) also is recommended for NSTEMI patients (with or without a stent). Thrombolytic therapy and antibiotics are not indicated for

Nonmodifiable Risk Factors

age gender ethnicity family history genetics. • Increasing age • Gender (highest incidence of CAD is among middle-aged men, but for men over age 45 and women over age 55, the risk for CAD increases for both genders) • Ethnicity (more common in white than black men) • Genetic predisposition and family history of heart disease Heart disease is often diagnosed about 10 years later in women than men. -possible reason may be the cardioprotective effects of estrogen -hormone replacement therapy does not protect women after menopause and may be harmful. Because women are older at the time of CAD diagnosis, they are more likely to have co-morbidities (e.g., hypertension, diabetes). Women are more likely than men to die after their first MI. CAD is present in black women at rates higher than their white or Hispanic counterparts. heart disease kills almost 10 times more women than breast cancer. It is the leading cause of death in women.

The nurse is caring for a patient who is 2 days post MI. The patient reports that she is experiencing chest pain when she takes a deep breath. Which action would be a priority? a. Notify the provider STAT and obtain a 12-lead ECG. b. Obtain vital signs and auscultate for a pericardial friction rub. c. Apply high-flow O2 by face mask and auscultate breath sounds. d. Medicate the patient with as-needed analgesic and reevaluate in 30 minutes.

b Rationale: Acute pericarditis is inflammation of the visceral and/or parietal pericardium. It often occurs 2 to 3 days after an acute myocardial infarction. Chest pain may vary from mild to severe. It is worsened by inspiration, coughing, and movement of the upper body. Sitting in a forward position often relieves the pain. The pain is usually different from pain associated with a myocardial infarction. Assessment of the patient with pericarditis may reveal a friction rub over the pericardium.

A hospitalized patient with a history of chronic stable angina tells the nurse that she is having chest pain. The nurse bases his actions on the knowledge that ischemia a. will always progress to myocardial infarction. b. can be relieved by rest, nitroglycerin, or both. c. is often associated with vomiting and extreme fatigue. d. indicates that irreversible myocardial damage is occurring.

b Rationale: Chronic stable angina is chest pain that occurs intermittently over a long period with the same pattern of onset, duration, and intensity of symptoms. The chest pain is relieved by rest or by rest and medication (e.g., nitroglycerin). The ischemia is transient and does not cause myocardial damage.

After teaching about ways to decrease risk factors for CAD, the nurse recognizes that further instruction is needed when the patient says a. "I can keep my blood pressure normal with medication." b. "I would like to add weight lifting to my exercise program." c. "I can change my diet to decrease my intake of saturated fats." d. "I will change my lifestyle to reduce activities that increase my stress."

b Rationale: Risk factors for coronary artery disease include high serum levels of lipids, high BP, tobacco use, physical inactivity, obesity, diabetes, metabolic syndrome, certain psychologic states, and high homocysteine levels. Weight lifting is not a cardioprotective exercise. An example of health-promoting regular physical activity is brisk walking (3 to 4 miles/hr) for at least 30 minutes 5 or more times each week.

1. In teaching a patient about coronary artery disease, the nurse explains that the changes that occur in this disorder include (select all that apply) a. diffuse involvement of plaque formation in coronary veins. b. abnormal levels of cholesterol, especially low-density lipoproteins. c. accumulation of lipid and fibrous tissue within the coronary arteries. d. development of angina due to a decreased blood supply to the heart muscle. e. chronic vasoconstriction of coronary arteries leading to permanent vasospasm.

b, c, d Rationale: Atherosclerosis is the major cause of coronary artery disease (CAD). It is characterized by a focal deposit of cholesterol and lipids, primarily within the intimal wall of the artery. The endothelial lining of the coronary arteries becomes inflamed from the presence of unstable plaques and the oxidation of low-density lipoprotein (LDL) cholesterol. Fibrous plaque causes progressive changes in the endothelium of the arterial wall. The result is a narrowing of the vessel lumen and a reduction in blood flow to the myocardial tissue.

The most common finding in people at risk for sudden cardiac death is a. aortic valve disease. b. mitral valve disease. c. left ventricular dysfunction. d. atherosclerotic heart disease.

c Rationale: Left ventricular dysfunction (ejection fraction less than 30%) and ventricular dysrhythmias after myocardial infarction are the strongest predictors of sudden cardiac death

A patient is recovering from an uncomplicated MI. Which rehabilitation guideline is a priority to include in the teaching plan? a. Refrain from sexual activity for a minimum of 3 weeks. b. Plan a diet program that aims for a 1- to 2-lb weight loss per week. c. Begin an exercise program that aims for at least 5 30-minute sessions per week. d. Consider the use of erectile agents and prophylactic NTG before engaging in sexual activity.

c Rationale: Physical activity should be regular, rhythmic, and repetitive, with the use of large muscles to build up endurance (e.g., walking, cycling, swimming, rowing). Physical activity sessions should be at least 30 minutes long. Teach the patient to begin slowly at personal tolerance (perhaps only 5 to 10 minutes) and build up to 30 minutes.

Angina

chest pain Clinical manifestation of myocardial ischemia. Caused by either an increased demand for O2 or a decreased supply of O2. Most common reason: significant narrowing of 1 or more coronary arteries by atherosclerosis. This leads to insufficient blood flow to the heart muscle. For ischemia to occur from an atherosclerotic plaque, the artery is usually blocked (stenosed) 70% or more (50% or more for the left main coronary artery).

atherosclerosis

comes from 2 Greek words: athere, meaning "fatty mush," skleros, meaning "hard." atherosclerosis begins as soft deposits of fat that harden with age. -why referred to as "hardening of the arteries." -can occur in any artery in the body. -When the atheromas (fatty deposits) form in the coronary arteries, the disease is called CAD. -terms arteriosclerotic heart disease (ASHD), cardiovascular heart disease (CVHD), ischemic heart disease (IHD), coronary heart disease (CHD), are other terms used to describe CAD. -are often used interchangeably. it is the major cause of CAD. characterized by lipid deposits within the intima of the artery. Endothelial injury and inflammation play a key role in the development of atherosclerosis.

Fatty Streak

earliest lesions of atherosclerosis -are lipid-filled smooth muscle cells -streaks of fat develop within the smooth muscle cells -yellow tinge appears can be seen in the coronary arteries by age 20 -involve an increasing amount of surface area as one ages Treatment that lowers low-density lipoprotein (LDL) cholesterol may slow this process

HMG-CoA Reductase Inhibitors (Statins)

most widely used lipid-lowering drugs -inhibit the synthesis of cholesterol in the liver. -result of inhibiting cholesterol synthesis is an increase in hepatic LDL receptors -allows the liver to remove more LDL from the blood -cause a minor increase in HDL and lower CRP levels -statins are not equal in reducing LDL levels. Rosuvastatin (Crestor) -most potent statin Serious adverse effects: -liver damage -myalgia (muscle ache or weakness without breakdown of skeletal muscle) that can progress to rhabdomyolysis (breakdown of skeletal muscle) Liver enzymes (ALT/AST) are initially monitored and rechecked with any increase in dosage. creatine kinase isoenzyme, CK-MM -found in skeletal muscle -assessed if symptoms of myopathy (e.g., muscle aches, weakness) occur.

C-reactive protein (CRP)

protein made by the liver -a nonspecific marker of inflammation -levels rise when there is systemic inflammation, such as rheumatoid arthritis or inflammatory bowel disease. CRP and levels of lipoprotein(a), which has pro-inflammatory properties, are increased in many patients with CAD CRP and lipoprotein(a) are not a part of a routine risk assessment for CAD -used in patients with an intermediate- or high-risk suspicion of CAD

Fibrous plaque

stage that is the beginning of progressive changes in the endothelium of the arterial wall -can appear in the coronary arteries by age 30 and increase with age. Normally, the endothelium repairs itself immediately. This does not occur in the person with CAD. LDLs and growth factors from platelets stimulate smooth muscle proliferation and thickening of the arterial wall. Once endothelial injury has taken place, lipoproteins (carrier proteins within the bloodstream) transport cholesterol and other lipids into the arterial intima. Collagen covers the fatty streak and forms a fibrous plaque with a grayish or whitish appearance. The result is a narrowing of the vessel lumen and reduced blood flow to the distal tissue

A patient with a history of asthma presents to the emergency department with wheezing, chest pain, and shortness of breath. Which type of medication listed in the patient's medication profile may be responsible for the patient's symptoms? Short-acting nitrate β-adrenergic blocker Calcium channel blocker Angiotensin II receptor blocker

β-adrenergic blocker The patient with a history of asthma should avoid β-adrenergic blockers because the side effects include wheezing from bronchospasm. Short-acting nitrates are the first-line treatment for a patient with angina and can be used safely by patients who have asthma. Calcium channel blockers are used in patients who have asthma if β-blockers are contraindicated, poorly tolerated, or do not control anginal symptoms. Angiotensin II receptor blockers can be given to the patient safely because they have no harmful effects on the respiratory system.

Diabetes teaching

• Follow the recommended diet. • Control or reduce weight. • Take prescribed drugs for diabetes. • Monitor blood glucose levels regularly and follow up with HCP regularly.

Psychologic state teaching

• Increase awareness of behaviors that are harmful to health. • Change patterns that add to stress (e.g., get up 30 min earlier so that breakfast is not eaten on way to work). • Set realistic goals for self. • Reassess priorities in light of identified risk factors. • Learn effective stress management strategies • Seek professional help if feeling depressed, angry, anxious, etc. • Plan time for adequate rest and sleep

Simvistatin (Zocor) DRUG ALERT

• Increased risk for rhabdomyolysis when used with fibric acid derivatives -gemfibrozil [Lopid]) -niacin (Niaspan -erythromycin. • Manifestations -high creatine kinase levels -muscle pain. • Prothrombin times -increase in patients taking warfarin (Coumadin).

Hypertension Teaching

• Monitor home-based BP and obtain regular checkups. • Take prescribed drugs for BP control. • Reduce salt intake. • Stop tobacco use. Avoid exposure to environmental tobacco (secondhand) smoke. • Control or reduce weight. • Perform physical activity daily.

Ezetimibe (Zetia)

selectively inhibits absorption of dietary and biliary cholesterol across the intestinal wall -adjunct to dietary changes, especially for patients with primary hypercholesterolemia. combined with a statin -ezetimibe and simvastatin (Vytorin]) -or as 2 separate drugs (ezetimibe and a statin) -greater reductions in LDLs occur.

Niacin (Niaspan)

water-soluble B vitamin -lowers LDL and triglyceride levels -interferes with their synthesis -at high doses, increases HDL levels better than many other lipid-lowering drugs Side effects: -severe flushing -pruritus -gastrointestinal (GI) symptoms -orthostatic hypotension • Flushing may occur: -within 20 mins after taking -especially of face and neck -lasts for 30 to 60 minutes. • Premedicate with aspirin or NSAID -30 minutes before taking -to reduce flushing. extended-release niacin may decrease side effects.

Tobacco use teaching

• Begin a tobacco cessation program. • Change daily routines associated with tobacco use to reduce desire to smoke. • Substitute other activities for smoking. • Ask caregivers to support efforts to stop smoking. • Avoid exposure to environmental tobacco smoke.

Obesity teaching

• Change eating patterns and habits. • Reduce caloric intake to achieve body mass index of 18.5 to 24.9 kg/m2. • Increase physical activity to increase caloric expenditure. • Avoid fad and crash diets, which are not effective over time. • Avoid large, heavy meals. Consider smaller, more frequent meals.

Developmental stages of CAD

Develop over the years. Stages: (1) fatty streak (2) fibrous plaque (3) complicated lesion.

High serum lipids teaching

• Reduce total fat intake. • Reduce saturated fat intake. • Take prescribed drugs for lipid reduction. • Adjust total caloric intake to achieve and maintain ideal body weight. • Engage in daily physical activity. • Increase amount of complex carbohydrates, fiber, and vegetable proteins in diet. • Follow-up with HCP for regular lipid panel assessments.

Gemfibrozil (Lopid)

• increase the risk for bleeding in patients taking warfarin (Coumadin) • Increases the risk of hypoglycemia in patients taking repaglinide (Prandin)


संबंधित स्टडी सेट्स

Pharmacology - Gastrointestinal Medications

View Set

Naturfag - Bigbang og Kapittel 6

View Set

Chapter 18: The French Revolution

View Set

Music Appreciation - Midterm Exam (Quiz 4+)

View Set

GSCM 410 Final Study Guide (CH#1-3)

View Set

Dave Ramsey Personal Finance Chapter 4: Debt

View Set

What is the best mode of transportation?

View Set

Section 2: Humans Try to Control Nature

View Set